Tumours Flashcards

1
Q

@# 6. A 60-year-old woman undergoing follow-up CT under the care of the oncologist develops a new expansile lytic lesion. Which of the following primary tumours usually causes an expansile lytic metastasis?

A. Cervix

B. Uterus

C. Ovary

D. Thyroid

E. Rectum

A

D. Thyroid

Renal cell carcinoma also causes expansile lytic metastases.

How well did you know this?
1
Not at all
2
3
4
5
Perfectly
2
Q
  1. Which of the following causes parallel spiculated (hair-onend) rather than divergent speculated (sunray) periosteal reaction?

A. Osteosarcoma

B. Ewing’s sarcoma

C. Sigmoid colon cancer metastasis

D. Hemangioma

E. Meningioma

A

B. Ewing’s sarcoma

Other causes include syphilis and infantile cortical hyperostosis.

How well did you know this?
1
Not at all
2
3
4
5
Perfectly
3
Q
  1. Which of the following is a cause of increased rather than decreased uptake on bone scan?

A. Cysts

B. Haemangioma

C. Leukemia

D. Spina bifida

E. Recent tooth extraction

A

E. Recent tooth extraction

Haemangioma and leukaemia as well as myeloma can show increased uptake as well.

How well did you know this?
1
Not at all
2
3
4
5
Perfectly
4
Q
  1. A 20-year-old man is referred for suspected malignant transformation of an osteochondroma. Which of the following is a cause for concern in an osteochondroma?

A. New lucency

B. Reduced scintigraphic activity

C. Growth plate before physeal closure

D. Asymptomatic nature

E. Cartilagenous cap > 0.5cm

A

A. New lucency

Concerning features for malignant transformation include new lucency, increased scintigraphic activity, growth after skeletal maturation, pain after puberty, cortical destruction and a cartilaginous cap > 1.5cm.

How well did you know this?
1
Not at all
2
3
4
5
Perfectly
5
Q

(Ped) 17. Which of the following is a feature of Ollier’s disease rather than Maffucci syndrome?

A. Associated with granulosa cell tumour of the ovary

B. Predilection for tubular bones, hands and feet

C. Very large enchondromas

D. Growth disturbance of long bones

E. Absence of haemangiomas

A

E. Absence of haemangiomas

Both are associated with juvenile granulosa cell ovarian tumours and other malignancies. The presence of multiple enchondromas with haemangiomas indicates Ollier’s disease.

How well did you know this?
1
Not at all
2
3
4
5
Perfectly
6
Q
  1. Which is the most common location for giant cell tumour?

A. Proximal tibia

B. Proximal femur

C. Distal femur

D. Proximal fibula

E. Patella

A

C. Distal femur

50-65% occur around the knee, but are rare in the patella.

How well did you know this?
1
Not at all
2
3
4
5
Perfectly
7
Q
  1. An osteoid osteoma is thought to be the cause of painful scoliosis in a 20-year-old man. Which of the following is the single best answer regarding osteoid osteomas?

A. The nidus appears sclerotic with surrounding lucency on CT

B. Most commonly found in long bones of the lower limbs

C. The nidus does not enhance on CT

D. Reactive sclerosis around the nidus is uncommon

E. The nidus demonstrates decreased activity on bone Scintigraphy

A

B. Most commonly found in long bones of the lower limbs

Although osteoid osteomas can occur in any bone, they are most common in the metadiaphyseal femur and tibia. The nidus appears lucent on radiographs, intensely active on bone scan – with surrounding well-defined luceny (double doughnut sign), isointense to muscle on T1 and variable SI on T2.

How well did you know this?
1
Not at all
2
3
4
5
Perfectly
8
Q

@# 24. Plain film, CT and MRI are performed for the investigation of suspected chordoma. Which is the best answer?

A. Radiographic appearances show sacral osteosclerosis

B. Coarse calcification often present with associated soft tissue

C. Areas of low attenuation within a mass on CT

D. Intermediate SI on T2

E. Arise from the spinal canal

A

C. Areas of low attenuation within a mass on CT

Chordoma usually appears as a low attenuation mass on CT.

How well did you know this?
1
Not at all
2
3
4
5
Perfectly
9
Q
  1. A 60-year-old man with several months’ history of back pain, worse when sitting, and with no bowel or bladder symptoms, undergoes evaluation with MRI. This shows a lobulated presacral mass, low SI on T1 with several areas of high SI within it, most likely to represent areas of calcification and haemorrhage. Which is the most probable diagnosis?

A. Chordoma

B. Chondrosarcoma

C. Myxopapillary ependymoma

D. Metastasis

E. Giant cell tumour

A

A. Chordoma

This is the most common primary malignant tumour of the sacrum.

Giant cell tumours are the second most common cause and are indistinguishable from chordomas on MRI.

How well did you know this?
1
Not at all
2
3
4
5
Perfectly
10
Q
  1. Which of the following indicates telangiectatic osteosarcoma (TOS) rather than an aneurismal bone cyst (ABC)?

A. Enhancing septa without nodularity on MR

B. Marked expansile remodelling of bone

C. Cortical thinning

D. Presence of osteoid matrix with septal regions on CT

E. Presence of haemorrhagic spaces

A

D. Presence of osteoid matrix with septal regions on CT

Thick peripheral septa with nodularity, presence of an osteoid matrix within nodular or septal regions, and aggressive growth features such as cortical destruction indicate TOS rather than ABC.

How well did you know this?
1
Not at all
2
3
4
5
Perfectly
11
Q
  1. Considering post-radiotherapy changes of soft tissue tumours on MRI:

A. Earliest radiation change is demonstrated by increased SI in marrow between 6-12 weeks

B. SI changes are due to replacement of marrow by fat

C. In most cases, complete replacement occurs in 12-14 weeks

D. Regeneration of normal marrow is common

E. Radiation field is usually poorly defined on MR

A

B. SI changes are due to replacement of marrow by fat

The earliest changes occur 1-6 weeks after therapy is initiated and is due to replacement of marrow by fat.

Complete replacement occurs within 6-8 weeks.

Due to high radiation doses used in soft tissue tumour treatment, regeneration of normal marrow is rare, but can occur in young patients.

The radiation field is usually well-defined.

How well did you know this?
1
Not at all
2
3
4
5
Perfectly
12
Q
  1. Considering soft tissue response to chemotherapy:

A. Neoadjuvant chemotherapy increases recurrence-free survival but not overall survival in high grade soft tissue sarcoma

B. Chemotherapy may cause a substantial increase in tumour size initially

C. Tumour size is the most accurate predictor of biologic response to tumour treatment

D. Contrast enhancement limits interpretation of intralesional necrosis

E. Intralesional haemorrhage post-chemotherapy is very rare

A

B. Chemotherapy may cause a substantial increase in tumour size initially

Treatment induced necrosis is a more accurate predictor of tumour response than size.

Tumour size can indeed increase initially due to intralesional haemorrhage.

The degree of necrosis is best evaluated with gadolinium enhanced T1 fat suppressed sequences.

Neoadjuvant chemotherapy increases both recurrence-free and overall survival.

How well did you know this?
1
Not at all
2
3
4
5
Perfectly
13
Q
  1. Which of the following favours chondroma rather than chondrosarcoma?

A. Size > 3cm

B. Pain

C. Age > 30

D. Location in hands and feet

E. Permeation into soft tissues

A

D. Location in hands and feet

Chondromas are usually < 3cm, painless and in younger patients in the peripheral skeleton.

How well did you know this?
1
Not at all
2
3
4
5
Perfectly
14
Q
  1. In a 19-year-old male with painful scoliosis, a well-defined 3cm geographic osteolytic lesion is seen in the right posterior seventh rib with slight expansion and sharp sclerotic margins. CT is performed and shows punctate calcification within the lesion and adjacent sclerotic bone. MRI shows lowintermediate T1 and intermediate-high T2 signal with bone marrow oedema. Which is the most likely diagnosis?

A. Giant cell tumour

B. Fibrous dysplasia

C. Enchondroma

D. Osteoblastoma

E. Aneurysmal bone cyst

A

D. Osteoblastoma

Osteoblastomas share clinical and histological features with osteoid osteomas and 3-12% occur in ribs.

How well did you know this?
1
Not at all
2
3
4
5
Perfectly
15
Q
  1. Regarding malignant fibrous histiocytoma:

A. Central mineralisation is common

B. Occur more often in bone than in soft tissue

C. Is the main primary malignant tumour of fibrous origin affecting bone

D. Most cases arise in Paget’s disease

E. Reactive changes are common

A

C. Is the main primary malignant tumour of fibrous origin affecting bone

Occur much more commonly in soft tissue than bone.

25% occur in pre-existing conditions such as Paget’s.

They usually present with insidious onset pain and swelling,

and central mineralisation and reactive changes are uncommon.

How well did you know this?
1
Not at all
2
3
4
5
Perfectly
16
Q
  1. Which of the following demonstrates the most uptake on PET/CT?

A. Primary bone lymphoma

B. Osteosarcoma

C. Chondrosarcoma

D. Enchondroma

E. Osteochondroma

A

A. Primary bone lymphoma

Greatest FDG uptake occurs in primary bone lymphoma and Ewing’s sarcoma.

Osteosarcoma demonstrates moderate uptake.

Most benign bone lesions are non-FDG avid, with the exception of high giant cell containing tumours (Giant Cell Tumors (GCT), osteoblastomas, aneurysmal bone cysts) and fibrous lesions (fibrous dysplasia).

How well did you know this?
1
Not at all
2
3
4
5
Perfectly
17
Q

1) A 34-year-old woman presents with pain and swelling of the right knee over the previous 2 months. Plain films demonstrate a wellcircumscribed, expansile, lytic lesion eccentrically located in the subarticular region of the right distal femur. The lesion has a narrow, non-sclerotic zone of transition. What is the most likely diagnosis?

a. giant cell tumour

b. enchondroma

c. fibrous cortical defect

d. fibrous dysplasia

e. aneurysmal bone cyst

A

a. giant cell tumour

The vast majority of giant cell tumours occur in patients with closed epiphyses, and although they may originate in the metaphysis, lesions typically involve the epiphysis and abut the subarticular surface.

They are classically eccentrically located lesions with a narrow zone of transition, no sclerosis, and no internal matrix mineralization.

Giant cell tumours tend to be locally aggressive, with a high recurrence rate after initial treatment.

Enchondromas are the commonest benign cystic lesion of the phalanges, though they are also seen in the long bones. However, those in the long bones almost always contain calcified chondroid matrix.

Aneurysmal bone cysts are often seen as an eccentric lytic expansile lesion, but patients are nearly all under the age of 30.

Monostotic fibrous dysplasia is more commonly seen in the proximal femur than distally, and lesions tend to have a sclerotic margin.

Fibrous cortical defects are asymptomatic lesions seen in children, which usually regress spontaneously, so they are only rarely seen after the age of 30. They typically appear as lytic lesions with a thin sclerotic border in the metaphysis of a long bone.

How well did you know this?
1
Not at all
2
3
4
5
Perfectly
18
Q

@# 12) A female adult patient with right shoulder pain is shown to have multiple markedly expansile lytic lesions within the scapula and clavicle secondary to metastatic malignant spread. Which of thefollowing is most likely to be the primary site of malignancy?

a. renal

b. breast

c. cervical

d. colon

e. bronchus

A

a. renal

The common cancers that typically metastasize to bone are breast, lung, thyroid, renal and prostate. Due to the high prevalence of colon cancer, even though only a relatively small proportion metastasizes to bone, it forms a significant proportion of bone metastases.

Prostatic metastases are typically sclerotic, whereas breast deposits are mixed.

Colonic bone metastases are usually lytic, with renal metastases typically lytic and expansile due to their highly vascular nature.

Other less frequent sources of lytic expansile metastases include thyroid, melanoma andphaeochromocytoma.

How well did you know this?
1
Not at all
2
3
4
5
Perfectly
19
Q

@# 13) A ‘fallen fragment’ seen within a lytic bone lesion is most commonly associated with which of the following?

a. aneurysmal bone cyst

b. unicameral (simple) bone cyst

c. giant cell tumour

d. eosinophilic granuloma

e. chondroblastoma

A

b. unicameral (simple) bone cyst

The fallen fragment is virtually pathognomonic for a simple bone cyst. It represents a fragment from a pathological fracture through the lesion, which has fallen to lie in a dependent location in the cyst matrix.

How well did you know this?
1
Not at all
2
3
4
5
Perfectly
20
Q

(CNS) 14) A 20-year-old man presents with gradual onset of neck pain and a painful lump in the upper neck posteriorly. Plain films show an apparent destructive lesion of the C2 vertebra. MRI shows a large lesion arising from the posterior elements of C2 and comprising multiple cysts with fluid–fluid levels, with preservation of the vertebral body. What is the most likely diagnosis?

a. aneurysmal bone cyst

b. giant cell tumour

c. chordoma

d. fibrous dysplasia

e. telangiectatic osteosarcoma

A

a. aneurysmal bone cyst

Aneurysmal bone cysts are seen mainly in patients under 20 years of age (75%) and affect the posterior elements when involving the spine. They may arise de novo, or secondary to another lesion such as a giant cell tumour (GCT) or fibrous dysplasia. Both GCTs and telangiectatic osteosarcomas may cause cysts with fluid–fluid levels on MRI, but GCTs arise from vertebral bodies and usually occur in the sacrum. Telangiectatic osteosarcomas usually affect long bones. Chordomas are malignant tumours that usually affect the vertebral body, with destruction and invasion of the discs and adjacent structures.

How well did you know this?
1
Not at all
2
3
4
5
Perfectly
21
Q

17) In imaging of focal bone lesions in the appendicular skeleton, which of the following radiographic features is most likely to indicate an aggressive or malignant process?

a. cortical expansion

b. lytic process

c. periosteal reaction

d. multiple lesions

e. wide zone of transition

A

e. wide zone of transition

The zone of transition relates to the interface between the tumour margin and the host bone. It is an extremely important discriminator, particularly for lytic lesions. Lesions with a well-defined margin (and therefore narrow zone of transition) are described as geographic and are usually non-aggressive, whereas those with a wide zone of transition are termed ‘permeative’ and are often malignant or aggressive (such as in osteomyelitis). Cortical expansion without destruction is seen in many benign or slow-growing conditions such as fibrous cortical defect and aneurysmal bone cyst. Many bone lesions, both benign and aggressive, are lytic. Periosteal reaction does not indicate an aggressive lesion as such, but the pattern of reaction can do so. Multiplicity is not an indicator of malignancy, as it can be seen in benign and self-limiting processes (such as multiple enchondromatosis and neurofibromatosis). Equally, a solitary lesion may be malignant.

How well did you know this?
1
Not at all
2
3
4
5
Perfectly
22
Q

19) The presence of punctate, ring-like or arcuate calcification in a lytic bone lesion on plain radiography is most commonly associated with which of the following matrix types?

a. osteoblastic

b. fibrous

c. cartilaginous

d. cellular

e. mixed

A

c. cartilaginous

Chondroid tumour matrix may or may not calcify, but, if it does, the pattern is characteristically in arcs or circles and is sometimes described as ‘popcorn’.

Osteoid matrix when calcified is usually dense and homogeneous like a cloud.

Calcified fibrous matrix has a characteristic ground-glass appearance,

whereas a cellular tumour usually does not show matrix calcification.

A mixed matrix will show mixed characteristics.

How well did you know this?
1
Not at all
2
3
4
5
Perfectly
23
Q

@# (Ped) 22) A 4-year-old boy is investigated for abnormal gait, with swelling and deformity of the right lower leg. Radiographs reveal epiphyseal irregularity and multiple abnormal ossifications around the medial portions of the distal femoral, and proximal and distal tibial epiphyses of the affected leg, with normal appearances of the lateral epiphyses and the whole of the contralateral lower limb. MRI demonstrates that the ossifications lie within the epiphyseal cartilage. What is the described condition?

a. hereditary multiple exostoses

b. Trevor’s disease

c. Ollier’s disease

d. Morquio’s syndrome

e. warfarin embryopathy

A

b. Trevor’s disease

Trevor’s disease (also called dysplasia epiphysealis hemimelica) is a rare developmental bone dysplasia.

It primarily occurs in children aged 2–4 years and affects boys more commonly than girls.

It shows a preponderance for the lower limbs, most commonly affecting the knee and ankle, and demonstrates single or multiple osteocartilaginous tumours arising from epiphyses.

The lesion is characteristically hemimelic, involving either the medial (two-thirds of cases) or lateral aspect of the ossification centres. Cases can be classified as localized, classic or generalized.

How well did you know this?
1
Not at all
2
3
4
5
Perfectly
24
Q

23) Plain radiographs of the hands in a young woman are performed for unilateral deformity. These show multiple lytic lesions in the medullary cavities of the tubular bones with cortical expansion and matrix mineralization, and associated Madelung deformity. The changes are unilateral. What is the most likely diagnosis?

a. Maffucci’s syndrome

b. Ollier’s disease

c. Trevor’s disease

d. Lichtenstein–Jaffe´ disease

e. Morquio’s syndrome

A

b. Ollier’s disease

Ollier’s disease or multiple enchondromatosis is characterized by the presence of benign intraosseous cartilaginous tumours. The estimated prevalence of the disease is 1 in 100 000. The distribution and number of lesions are variable, but are often unilateral and monomelic. Complications include pain, skeletal deformities, limb length discrepancy (including Madelung’s deformity) and the potential risk of malignant change to chondrosarcoma in 20–50% of cases. The condition in which enchondromas are associated with haemangiomas is known as Maffucci’s syndrome. Neither is usually inherited.

Trevor’s disease is an epiphyseal dysplasia,

whereas Lichtenstein–Jaffe´ disease is another name for fibrous dysplasia.

Morquio’s syndrome is one of the lysosomal storage disorders known as the mucopolysaccharidoses.

How well did you know this?
1
Not at all
2
3
4
5
Perfectly
25
Q

24) Of the following types of periosteal reaction, select the one most likely to indicate a benign process?

a. soap-bubble

b. sunray

c. hair-on-end

d. laminated

e. Codman’s triangle

A

a. soap-bubble

Periosteal reactions are usually a radiographic manifestation of underlying bone disease.

The term ‘soap bubble’ refers to expansion of the cortex without destruction by a lytic bone lesion. The intact cortex usually indicates a benign process, whereas cortical destruction is associated with malignant or aggressive lesions.

Sunray and hair-on-end reactions are spiculated forms of periosteal reaction that occur following periosteal elevation by tumour, with tumour preventing the subperiosteal space from filling with new bone.

Laminated or ‘onion-skin’ reaction occurs with both malignant and benign processes and indicates an intermittent or cyclical process.

Codman’s triangles are formed by elevation and then destruction of the periosteum. They are usually related to malignant tumours but can also be formed by aggressive benign processes.

How well did you know this?
1
Not at all
2
3
4
5
Perfectly
26
Q

35) A 19-year-old man presents to his general practitioner with a sudden onset of painful scoliosis. His pain improves with prescribed aspirin while awaiting MRI. MRI reveals a localized area of inflammatory change in the left pedicle of L1. Subsequent CT shows marked sclerosis in the same region with a 5 mm, cortically based central lucency. What is the most likely cause?

a. plasma cell cytoma

b. osteosarcoma

c. osteoid osteoma

d. Brodie’s abscess

e. lymphoma

A

c. osteoid osteoma

Osteoid osteoma accounts for 12% of benign neoplasms of bone. It is most commonly located in the cortex of long bones (50% in the femur and tibia) with 15% in the spine, typically the pedicle. It rarely exceeds 15 mm in size. Young men are most commonly affected, with pain as the predominant presenting feature due to the extensive inflammatory reaction and vascularity of the lesion. With spinal lesions this results in a painful positional scoliosis, though the majority of patients experience improvement of the pain with salicylates. The lucent central area or nidus represents the underlying pathological process, with the surrounding sclerosis representing reactive inflammatory change in normal bone. Treatment traditionally was surgical curettage, but radiologically guided percutaneous radiofrequency ablation is now used.

How well did you know this?
1
Not at all
2
3
4
5
Perfectly
27
Q

41) A 21-year-old man presents with right hip pain. He has a history of Ewing’s sarcoma of the right hemi-pelvis when aged 11, which was treated with limb-sparing surgery and chemoradiotherapy. Plain radiography shows well-defined regional sclerosis, and isotope bone scan demonstrates a corresponding photopenic area. What is the most likely cause of his pain?

a. recurrent Ewing’s sarcoma

b. heterotopic ossification

c. radiation necrosis

d. osteoarthritis

e. osteosarcoma

A

c. radiation necrosis

Ewing’s sarcoma is a relatively common malignant bone marrow tumour related to, and sharing a common chromosomal translocation with peripheral neuroectodermal tumours. It is a very aggressive tumour thatis expressed in the radiological findings of permeative osteolysis, cortical erosion, periostitis and a soft-tissue mass. It principally affects the lower half of the skeleton, with the most frequent location being metadiaphyseal in the femur, ilium, tibia, humerus, fibula and ribs. Both radio- and chemotherapy are used in treatment. Sclerosis within several months of treatment usually indicates bone healing or disease recurrence/persistence. Radiation-induced osteonecrosis is mainly an effect on the osteoblasts and is dose dependent (deterministic). It may be seen within the mandible within 1 year, but in other sites the latent period is longer and can be a number of years.

How well did you know this?
1
Not at all
2
3
4
5
Perfectly
28
Q

42) On MRI performed for a tender osteochondroma of the femoral metaphysis in an adult, which feature is most useful in determining the presence of malignant change?

a. thickness of the cartilage cap

b. lesion size

c. compression of local nerves

d. fracture of the stalk

e. bursa formation

A

a. thickness of the cartilage cap

Osteochondromas are the commonest bone tumours and are considered developmental exostoses rather than true neoplasms. They represent 20–50% of benign and 10–15% of all bone tumours. They are made up of cortical and medullary bone and an overlying cartilage cap. The cortex and medulla of the osteochondroma are continuous with the underlying host bone. They are typically orientated away from an adjacent distal joint. Lesions are frequently solitary, but multiple lesions are seen in hereditary multiple exostoses, an autosomal dominant syndrome. Malignant transformation occurs in 1% of solitary lesions and in 3–5% of patients with hereditary multiple exostoses. After skeletal maturity, continued lesion growth, particularly of the cartilage cap, is suggestive of malignant transformation. Although benign lesions may reach 10 cm in size, the cartilage cap should not exceed 1.5 cm after skeletal maturation. Any bone that develops by enchondral ossification may develop an osteochondroma, the long bones of the lower extremity being most frequently affected.

How well did you know this?
1
Not at all
2
3
4
5
Perfectly
29
Q

@# 53) Of the following subtypes of osteosarcoma, which is associated with the most favourable 5-year survival?

a. multicentric

b. periosteal

c. paraosteal

d. telangiectatic

e. soft-tissue

A

c. paraosteal

Osteosarcoma is the second most common primary malignancy of bone after multiple myeloma, accounting for 15% of all primary bone tumours. It usually affects those aged 10–30.

Ninety-five per cent are of the primary osseous type and, of these, paraosteal osteosarcoma has the most favourable 5-year survival rate of 80%.

Other osteosarcomas of the primary osseous type include periosteal (5-year survival rate 50%) and telangiectatic (less than 20%).

Multicentric refers to synchronous osteoblastic osteosarcomas at multiple sites. It occurs exclusively in children aged 5–10, and carries an extremely poor prognosis.

The soft-tissue type is rare, representing only 1.2% of all soft-tissue tumours. These lesions are primary soft-tissue tumours with no attachment to bone. Death occurs within 3 years in the majority of cases, tumour size being the major predictor of outcome.

How well did you know this?
1
Not at all
2
3
4
5
Perfectly
30
Q

@# 54) A 30-year-old woman undergoes plain radiographic imaging of the hand for a palpable, painful hard lump on the dorsum. Plain radiographs show a well-defined bony mass applied closely to the diaphysis of the second metacarpal. CT shows a wide-based pedunculated lesion with a perpendicular orientation to the diaphysis, no cartilage cap and a matrix of mature trabeculated bone. What is the most likely diagnosis?

a. osteochondroma

b. multiple osteocartilaginous exostoses

c. bizarre paraosteal osteochondromatous proliferation

d. Codman’s tumour

e. dysplasia epiphysealis hemimelica

A

c. bizarre paraosteal osteochondromatous proliferation

Bizarre paraosteal osteochondromatous proliferation (also known as Nora’s lesion) is a rare condition usually seen in adults in the third and fourth decades of life. Osteochondroma-like lesions are seen most commonly at the proximal and middle phalanges, followed by the metacarpals and metatarsals. A relationship to trauma has been suggested but not proven. Other locations that may be affected include the long bones (especially those of the upper extremity), skull and jaw. It is thought to be a similar process to that which gives rise to lesions in myositis ossificans, reactive periostitis and subungual exostosis. On plain radiographs, a well-defined bony mass is seen attached to the surface of the parent bone.

Features differentiating this from osteochondroma are the absence of angulation away from the nearby physis and a wide base.

How well did you know this?
1
Not at all
2
3
4
5
Perfectly
31
Q

59) A middle-aged woman, known to suffer from polyostotic fibrous dysplasia, presents with a palpable, 3 cm, soft-tissue mass in the upper left thigh. MRI shows a relatively homogeneous, smooth, well-defined lesion located in an atrophic quadriceps muscle, which returns low signal on T1W images and high signal on T2W images. Following administration of intravenous gadolinium, the lesion shows moderately intense heterogeneous enhancement. What is the most likely pathological nature of the soft-tissue lesion?

a. soft-tissue myxoma

b. malignant fibrous histiocytoma

c. soft-tissue cavernous haemangioma

d. multiple lipomatosis

e. rhabdomyosarcoma

A

a. soft-tissue myxoma

The association of fibrous dysplasia and soft-tissue myxoma is well established and is commonly termed Mazabraud’s syndrome. The key is identifying the relationship between the bone and soft-tissue pathology, with the osseous features of fibrous dysplasia usually preceding the formation of a soft-tissue mass. The condition is non-familial and more commonly affects women, the thigh being the most common location. Typical MRI appearances are of a well-defined lesion with signal intensity similar to water, and often a fat rind or adjacent muscle high signal on T2W images is seen. Although uncommon, there have been reported cases of malignant change into osteosarcoma.

How well did you know this?
1
Not at all
2
3
4
5
Perfectly
32
Q

78) A 30-year-old man undergoes MRI of the whole of the left lower limb and pelvis for a mid-femoral destructive lytic lesion identified on radiography that is thought to represent a primary bone tumour. MRI shows that the disease is confined to the femur with a 5 cm diaphyseal lesion and a 1 cm proximal metaphyseal skip lesion. No enlarged lymph nodes are identified. CT scans of the chest, abdomen and pelvis show two metastatic nodules in the lower lobe of the left lung. Subsequent biopsy confirms the diagnosis of osteosarcoma. The cancer is correctly staged as which of the following?

a. T1 N0 M0

b. T1 N0 M1a

c. T2 N0 M1b

d. T3 N0 M1a

e. T3 N0 M1b

A

d. T3 N0 M1a

Complete staging of primary bone sarcomas is unusual in that it also incorporates the histological staging once tissue diagnosis via biopsy or surgical resection is available. The local TNM classification is T1 (single lesion less than 8 cm), T2 (single lesion over 8 cm) or T3 (skip lesions of any size). Nodal staging is N0 (no nodes) or N1 (any number of metastatic nodes). Metastatic spread is staged, accordingly, as M0 (no metastases), M1a (metastases to lung) or M1b (any other distant site). Once histology is available, tumours can be staged I–IV.

How well did you know this?
1
Not at all
2
3
4
5
Perfectly
33
Q

(Ped) 90) An 8-year-old boy with a 3-month history of increasing chest wall pain presents with a tender lump on the chest wall. Imaging of the chest shows a large inhomogeneous mass arising from a rib with a large intrathoracic component and preservation of tissue planes. There is associated rib destruction and a lamellated periosteal reaction is seen. What is the most likely diagnosis?

a. Ewing’s sarcoma

b. neuroblastoma

c. Hodgkin’s disease

d. osteomyelitis

e. hamartoma of chest wall

A

a. Ewing’s sarcoma

Ewing’s sarcoma is the commonest malignant bone tumour in children, and the ribs are involved in 30% of cases in children under 10. It is the commonest malignant chest wall tumour. Neuroblastoma presents in children under 5 as a well-defined, soft-tissue mass that may calcify and erodes/splays the ribs. Hodgkin’s disease presents in young adults, and usually involves bones by secondary involvement with direct invasion of sternum or ribs. Osteomyelitis presents at any age with a shorter history (usually less than 2 weeks), and imaging shows rib destruction with a relatively small mass and loss of tissue planes. Hamartoma of the chest wall presents in the first year of life as an extrapleural mass, causing partial/complete destruction of adjacent ribs. Significant calcification and compression of the adjacent lung occur.

How well did you know this?
1
Not at all
2
3
4
5
Perfectly
34
Q

91) A 70-year-old man undergoes CT of the skull for investigation of clinically apparent macrocephaly confirmed on skull radiography.You are asked by the referring clinician to review the images. Which finding is most likely to support a diagnosis of fibrous dysplasia over Paget’s disease?

a. widened diploe¨

b. asymmetrical involvement of the skull

c. sparing of the paranasal sinuses

d. osteoporosis

e. ground-glass medulla

A

e. ground-glass medulla

A ground-glass appearance is characteristic of fibrous dysplasia and is the most useful discriminating factor. Other features of fibrous dysplasia of the skull that can help distinguish it from Paget’s disease are symmetry of distribution, presence of a soft-tissue mass, cyst-like changes, thickness of the cranial cortices, and involvement of the paranasal sinuses, maxilla, sphenoid, orbits and nasal cavity.

How well did you know this?
1
Not at all
2
3
4
5
Perfectly
35
Q

93) An elderly man undergoes 99mTc-labelled diphosphonate bone scintigraphy. There is no uptake of tracer in the soft tissues, urinary tract or appendicular skeleton, but the axial skeleton shows diffuse homogeneous tracer uptake. No focal lesions are seen. What is the most likely cause of these appearances?

a. prostatic metastases

b. renal osteodystrophy

c. Paget’s disease

d. mastocytosis

e. myelofibrosis

A

a. prostatic metastases

The resulting pattern following diffuse uptake of 99mTc-labelled diphosphonates in the axial skeleton, with little or no uptake of tracer in the soft tissues or urinary tract, is frequently referred to as a superscan. When there is little uptake in the limbs, the cause is most likely to be diffuse metastases in the axial skeleton, most commonly prostatic or breast. Uptake in metabolic bone disease is more uniform in appearance, and extends into the distal appendicular skeleton. Intense calvarial uptake disproportionate to that in the remainder of the skeleton may also be seen. The most important factor is to recognize the scan as abnormal in the absence of focal lesions. The lack of renal uptake (absent kidney sign) is a useful discriminator.

How well did you know this?
1
Not at all
2
3
4
5
Perfectly
36
Q

100) Plain radiographs of the femur performed for pain reveal a centrally located lucent lesion in the medulla with a partially calcified matrix. Which of the following features favours a diagnosis of chondrosarcoma over enchondroma?

a. arc-and-ring matrix calcification

b. ground-glass matrix

c. multiple lesions

d. deep endosteal scalloping

e. lesion size over 5 cm

A

d. deep endosteal scalloping

Distinction of enchondroma and intramedullary chondrosarcoma in the appendicular skeleton proximal to the metacarpals/-tarsals is difficult radiologically. A series of 187 patients showed that chondrosarcoma was associated with endosteal scalloping, with scalloping involving more than two-thirds of the extent of the lesion being strongly suggestive of malignancy. Other powerful discriminating factors identified as favouring chondrosarcoma were cortical destruction, soft-tissue mass, periosteal reaction, radionuclide uptake at scintigraphy and pain associated with the lesion. Chondrosarcoma also tended to be larger with a mean size of 10 cm compared with 6.7 cm for enchondroma. A ground-glass matrix with arcuate calcification is characteristic of both types of cartilaginous lesion. Multiple lesions may be seen in both malignancy and enchondromatosis (Ollier’s disease).

How well did you know this?
1
Not at all
2
3
4
5
Perfectly
37
Q

1 A plain radiograph reveals a well-defined lucent lesion within the metaphysic and epiphysis of the distal femur. There is eccentric expansion and the cortex is thin but intact. It does not reach the articular surface. CT reveals fluid-fluid levels. Which of the following is the most likely cause?

(a) Giant cell tumour

(b) Aneurysmal bone cyst

(c) Enchondroma

(d) Non-ossifying fibroma

(e) Chondromyxoid fibroma

A

(b) Aneurysmal bone cyst

Of the conditions that typically cause lucent, eccentrically expanded lesions, only GCTs and ABCs have fluid-fluid levels on CT imaging. As GCTs do not reach the articular surface, this is most likely to be an ABC.

How well did you know this?
1
Not at all
2
3
4
5
Perfectly
38
Q

12 A 27 year old undergoes a CXR as part of an occupational assessment. The CXR shows a solitary expansile right 5th posterior rib mass. CT confirms the mass is arising from the rib and that it shows no aggressive features.
What is the likeliest diagnosis?

(a) Exostosis of the rib

(b) Benign cortical defect

(c) Langerhans cell histiocytosis

(d) Fibrous dysplasia

(e) Osteoblastoma

A

(d) Fibrous dysplasia

Fibrous dysplasia is the commonest benign rib tumour. Second most common is osteochondroma/bony exostosis but this typically occurs at the costochondral junction. Other benign possibilities include GCT and aneurysmal bone cyst.

How well did you know this?
1
Not at all
2
3
4
5
Perfectly
39
Q

13 A calcified mass is seen on a plain radiograph of a young man’s femur. The mass appears to be centred within the soft tissues of the thigh and the calcification is more prominent on the periphery of the mass. There is a radiolucent zone separating the lesion from the underlying bone, the cortex of which appears unaffected. Which of the following is the most likely diagnosis?

(a) Myositis ossificans

(b) Parosteal osteosarcoma

(c) Juxtacortical chondroma

(d) Osteochondroma

(e) Rhabdomyosarcoma

A

(a) Myositis ossificans

Myositis ossificans is a form of heterotopic bone formation within skeletal muscle, usually resulting from blunt trauma. Although parosteal osteosarcoma can have similar appearances, myositis ossificans typically has denser calcification in the periphery; osteosarcoma shows the reverse phenomenon, with denser calcification centrally. Juxtacortical chondroma typically scallops the underlying cortex. An osteochondroma is continuous with the underlying bone.

How well did you know this?
1
Not at all
2
3
4
5
Perfectly
40
Q

23 A 7 year old boy is brought to the GP by his parents, having noticed soft, blue-coloured growths on his right hand. The hand X-ray reveals multiple enchondromas. Which of the following features would confirm Mafucci’s syndrome as the diagnosis rather than Ollier’s disease?

(a) A first degree relative also affected

(b) Bilateral, predominantly symmetrical disease

(c) A discrepancy in arm length

(d) Sarcomatous degeneration

(e) Soft tissue haemangiomas

A

(e) Soft tissue haemangiomas

Both conditions describe multiple enchondromas affecting the hands and/or feet, and both tend to be unilateral; neither has a genetic component. Both conditions can lead to shortening of the involved arm/leg, resulting in length discrepancy. Malignant degeneration can be to osteosarcoma (young adults), or chondro/fibrosarcoma (older patients); it is more common in Mafucci’s syndrome, but is still seen in 5-30% of cases of Ollier’s disease. Maffucci’s syndrome describes enchondromas with additional multiple soft tissue haemangiomas, if bilateral there is marked asymmetry.

How well did you know this?
1
Not at all
2
3
4
5
Perfectly
41
Q

35 A 50 year old man presents with knee pain. Plain raqiographs show an 8 cm lytic lesion within the distal femoral metaphysis with endosteal scalloping and cortical thickening. CT shows matrix mineralisation.
Which of the following features does not favour a diagnosis of chondrosarcoma over enchondroma?

(a) The patient’s age

(b) The patient’s sex

(c) The lesion size

(d) The lesion site

(e) The CT findings

A

(e) The CT findings

Enchondromas present in a slightly younger age group and are more common in females. They typically affect the bones of the hands and feet and are usually less than 5 cm in size. Although both lesions show matrix mineralisation, this feature is slightly commoner in enchondromas. Other features to favour chondrosarcoma over enchondroma include presentation with a mass, cortical destruction and the presence of a soft tissue mass.

How well did you know this?
1
Not at all
2
3
4
5
Perfectly
42
Q

44 A 38· year old woman presents with a palpable lump in her thigh. Plain films show a lobulated ossified mass lying posterior to the femur with a connection to the cortex. The centre of the lesion is denser than the periphery. MRI shows a large associated soft tissue component. What is the likeliest diagnosis?

(a) Myositis ossificans

(b) Osteochondroma

(c) Parosteal osteosarcoma

(d) Periosteal osteosarcoma

(e) Extraosseous osteosarcoma

A

(c) Parosteal osteosarcoma

Parosteal osteosarcomas have the best prognosis of all osteosarcomas. If no stalk can be clearly identified they can be distinguished from myositis ossificans by the relative density of the centre of the ossified part of the lesion.

How well did you know this?
1
Not at all
2
3
4
5
Perfectly
43
Q

47 A 40 year old man presents with heel pain. Lateral radiograph of the foot reveals a well-defined 3 cm lesion located between the anterior and middle thirds of the calcaneus. The lesion is radiolucent with a thin rim of sclerosis and central calcification. What is the likely diagnosis?

(a) Desmoplastic fibroma

(b) Giant cell tumour

(c) lntraosseous lipoma

(d) Osteoid osteoma

(e) Unicameral bone cyst

A

(c) lntraosseous lipoma

lntraosseous lipomas are rare bone tumours; they are often asymptomatic and present incidental, but can be associated with pain. The commonest locations are within the proximal femur (Ward’s triangle) and in the area of the calcaneus described. In these areas there is a relative paucity of trabecular bone and it is thought that this leads to an ‘overshoot’ phenomenon during the transition of haematopoietic to fatty marrow, with the resultant formation of the lipoma. Central or ring calcification in a lucent lesion in this location of the calcaneus is said to be pathognomonic of an intraosseous lipoma and allows its distinction from a UBC. If clinical doubt persists MR imaging can be used for further clarification and to confirm the presence of fat.

How well did you know this?
1
Not at all
2
3
4
5
Perfectly
44
Q

48 Regarding osteoblastomas, which of the following is true?

(a) An expansile appearance on plain film implies malignancy

(b) Matrix calcification is a common plain film feature

(c) They result in scoliosis more commonly than osteoid osteoma ‘

(d) They rarely grow beyond 2 cm

(e) They usually present in the 6th-7th decades

A

(b) Matrix calcification is a common plain film feature

Osteoblastomas are uncommon primary bone tumours (< 1 %). 90% occur in 2nd - 3rd decades, although cases have been documented up to 72 years. They are histologically similar to osteoid osteoma but less well organised and by definition larger (> 2 cm). The majority (30- 40%) of cases occur in the spine, with a slight predominance for the posterior elements (55%). An expansile lesion in osteoblastoma is not typically associated with malignancy. Scoliosis can occur in both and is typically painful, but occurs more commonly in osteoid osteoma, where the scoliosis results from muscle spasm secondary to the inflammatory mediators produced. Osteoblastomas are not radiosensitive and surgical excision is performed in most cases, however, recurrence is seen in up to 50%.

How well did you know this?
1
Not at all
2
3
4
5
Perfectly
45
Q

50 A 3 month old infant presents with tender, hard swellings over a number of bones. Radiographs reveal a cortical hyperostosis and marked, diffuse, symmetrical periosteal reaction of the clavicles, ribs and mandible. Which of the following is the most likely diagnosis?

(a) Rickets

(b) Caffey’s disease

(c) Hypothyroidism

(d) Scurvy

(e) Ulcerative colitis

A

(b) Caffey’s disease

The appearances are typical of Caffey’s disease, also known as infantile cortical hyperostosis. Scurvy and rickets are unlikely to produce this picture in those < 6 months old.

How well did you know this?
1
Not at all
2
3
4
5
Perfectly
46
Q

1 You review a plain radiograph of the left femur of a 16 year old patient and note a lesion with a wide zone of transition and a marked ‘sunburst’ periosteal reaction. Which of the following is the most likely diagnosis?

(a) Aneurysmal bone cyst

(b) Ewing’s sarcoma

(c) Osteosarcoma

(d) Fibrosarcoma

(e) Osteoid osteoma

A

(c) Osteosarcoma

Of the above list, osteosarcoma is the most likely. The periosteal reaction (and wide zone of transition) indicates an aggressive lesion. Ewing’s sarcoma is more common in the 1-10 year old age group, and fibrosarcoma in the 30-60 year old age group.

How well did you know this?
1
Not at all
2
3
4
5
Perfectly
47
Q

8 A 23 year old woman complains of anterior chest wall pain. CXR shows a large expansile mass arising from the left 3rd rib. CT confirms the mass is solitary and demonstrates whorled areas of ‘rings and arcs’ calcification with no soft tissue component. Which of the following is the most likely diagnosis?

(a) Osteosarcoma

(b) Osteoblastoma

(c) Chondrosarcoma

(d) Askin tumour

(e) Plasmacytoma

A

(c) Chondrosarcoma

Rings and arcs of calcification indicate a chondroid matrix. Chondrosarcoma is the likeliest primary rib tumour in this age group. Other primary malignant tumours include plasmacytoma and lymphoma.

How well did you know this?
1
Not at all
2
3
4
5
Perfectly
48
Q

19 A plain radiograph of a 20 year old’s knee shows a well defined lesion in the distal femoral epiphysis. It is lytic, with a narrow zone of transition and a well defined sclerotic rim. The lesion does not reach the articular surface, and is not expansile. Which of the following conditions is the most likely?

(a) Aneurysmal bone cyst

(b) Chondroblastoma

(c) Giant cell tumour

(d) Simple bone cyst

(e) Metastasis

A

(b) Chondroblastoma

Chondroblastomas almost invariably occur in the epiphyses, and 90% occur in those under 30 years old. Lesions to consider in this location and age group include: infection, chondroblastomas, GCTs and, less commonly, ABCs and eosinophilic granulomas. With regards to the other conditions mentioned: GCTs abut the articular surface; ABCs are almost always expansile; simple bone cysts are not epiphyseal; metastases are unlikely in a 20 year old.

How well did you know this?
1
Not at all
2
3
4
5
Perfectly
49
Q

23 A 15 year old girl presents with a history of several months leg pain, worse at night which is relieved by salicylates. Plain films show a 1.3 cm lesion in the proximal tibial shaft which has a round central lucency with surrounding sclerosis. What is the likeliest diagnosis?

(a) Osteosarcoma

(b) Brodie abscess

(c) Aneurysmal bone cyst

(d) Osteoid osteoma

(e) Stress fracture

A

(d) Osteoid osteoma

The classical finding is that of a central, usually lucent nidus of less than 1.5 cm in size with varying degrees of surrounding sclerosis. There is often a typical clinical picture which enables differentiation from a Brodie abscess.

How well did you know this?
1
Not at all
2
3
4
5
Perfectly
50
Q

31 A patient has widespread metastatic disease. The skeletal metastases are sclerotic. Which of the following primary cancers most commonly produce sclerotic metastases?

(a) Renal cell carcinoma

(b) Carcinoid

(c) Wilm’s tumour

(d) Ovarian carcinoma

(e) Melanoma

A

(b) Carcinoid

Sclerotic metastases are typically the result of: prostate carcinoma, carcinoid, transitional cell carcinoma, breast (mixed), medulloblastoma, colon (occasionally), and lymphoma.

How well did you know this?
1
Not at all
2
3
4
5
Perfectly
51
Q

37 A 32 year old woman presents after minor trauma with a lump over the midshaft of her left tibia. Plain films show an 8 cm multilocular, slightly expansile, predominantly lytic lesion with a sclerotic margin, and a narrow zone of transition. The lesionis orientated longitudinally along the anterior tibial diaphysis. What is the most likely diagnosis?

(a) Adamantinoma

(b) Eosinophilic granuloma

(c) Fibrous dysplasia

(d) Non-ossifying fibroma

(e) Chondro/nyxoid fibroma

A

(a) Adamantinoma

These are locally aggressive lesions which tend to recur and after several recurrences can metastasize to lungs. The main differential diagnosis for this ‘soap-bubble’ like appearance is fibrous dysplasia, but this usually presents in a slightly younger age group.

How well did you know this?
1
Not at all
2
3
4
5
Perfectly
52
Q

43 A 70 year old woman presents with a longstanding right sided painless swelling on her back. US shows a lesion with an array of linear strands against an echogenic background. MRI shows the lesion lies between the posterior chest wall and the inferomedial bor£1er of the scapula. It is well defined, has signal characteristics similar to surrounding skeletal muscle and has linear areas of high signal. It enhances heterogeneously after the administration of i. v. gadolinium. What is the likeliest diagnosis?

(a) Liposarcoma

(b) Extra-abdominal desmoid

(c) Metastasis

(d) Elastofibroma

(e) Lipoma

A

(d) Elastofibroma

Elastofibroma is benign tumour forming as a reaction to mechanical friction most commonly seen in the subscapular area. Recent evidence suggests a genetic component. It is bilateral in 25%. Its site, presentation and imaging characteristics are usually typical, however, excision is still performed if there is any doubt with regards to malignant transformation.

How well did you know this?
1
Not at all
2
3
4
5
Perfectly
53
Q

52 A 31 year old previously well woman presents with pain just above the knee. Plain radiographs show a fracture through a well circumscribed, expansile, solitary lytic bone lesion with a narrow zone of transition. The lesion involves the’·articular surface but does not extend into the joint space. What is the likeliest diagnosis?

(a) Chondroblastoma

(b) Fibrous dysplasia

(c) Giant cell tumour

(d) Eosinophilic granuloma

(e) Aneurysmal bone cyst

A

(c) Giant cell tumour

This is the characteristic patient age and site of GCT, where the unfused physis acts as a barrier to spread of the lesion. After fusion, extension to within 1 cm of the articular surface is the commonest pattern, although transarticular spread has been reported. It can have either a ‘soap bubble’ like appearance or, as described here, a uniform lytic appearance. 10% present with pathological fractures.

How well did you know this?
1
Not at all
2
3
4
5
Perfectly
54
Q

55 A 3 year old child of short stature is found to have numerous bony exostoses on plain radiography. Which of the following statements is not true?

(a) Hereditary transmission is autosomal dominant

(b) Malignant transformation to osteosarcoma occurs in 1 %

(c) The distal femur and proximal tibia are most commonly involved bones

(d) It is more common in males

(e) There is an association with polydactyly

A

(b) Malignant transformation to osteosarcoma occurs in 1 %

Hereditary multiple exostoses (also known as diaphyseal aclasia) is an autosomal dominant condition characterised by multiple exostoses. Malignant transformation to chondrosarcoma occurs in 1-5%.

How well did you know this?
1
Not at all
2
3
4
5
Perfectly
55
Q

61 A 10 year old girl presents with precocious puberty and café au lait spots. McCune-Albright syndrome is suspected. Which of the following features would you not expect to see?

(a) A ground glass lesion in the medullary cavity of the femur

(b) Champagne glass appearance of the pelvis

(c) Endosteal scalloping of the femur with intervening normal cortex

(d) Cortical expansion of the ribs

(e) Increased uptake of a rib lesion on bone scintigraphy

A

(b) Champagne glass appearance of the pelvis

McCune-Albright syndrome is a form of fibrous dysplasia. It is seen in 10% of cases. Radiologically, it is characterised by a polyostotic, unilateral pattern of involvement. The skull is commonly involved. A champagne glass appearance of the pelvis is seen in achondroplasia.

How well did you know this?
1
Not at all
2
3
4
5
Perfectly
56
Q

66 A 20 year old man presents with a swelling in his thigh. He recalls innocuous trauma at this site a few weeks earlier. Plain films suggest a soft tissue mass with peripheral calcification at the level of the mid femur with a radiolucent zone separating the lesion from cortex. MRI shows a heterogeneous, well defined soft tissue mass, isointense to muscle on T1W and hyperintense on T2W, with curvilinear peripheral ar’e as of low signal intensity.
Which of the following is the likeliest diagnosis?

(a) Myositis ossificans

(b) Parosteal sarcoma

(c) Tumoural calcinosis

(d) Rhabdomyosarcoma

e) Chondrosarcoma

A

(a) Myositis ossificans

These are the typical findings of myositis ossificans. Although· in the acute stages, it can be confused with other entities, as it matures and calcifies, it can be discriminated from osteosarcomas by the pattern of peripheral calcification and from parosteal sarcomas by the lack of a connecting stalk to the cortex.

How well did you know this?
1
Not at all
2
3
4
5
Perfectly
57
Q

71 A 62 year old man presents with a history of a lump in his thigh which he feels has enlarged over several months. On plain films a soft tissue mass with punctuate calcification and adjacent cortical erosion is seen. At MRI the mass is confirmed to be in the soft tissues, is heterogeneous in signal intensity but is generally isointense to muscle on T1W and hyperintense on T2W. What is the likeliest diagnosis?

(a) Parosteal sarcoma

(b) Liposarcoma

(c) Lipoma

(d) Malignant fibrous histiocytoma

(e) Rhabdomyosarcoma

A

(d) Malignant fibrous histiocytoma

MFH is the commonest primary malignant soft tissue tumour of later life. It presents as a painless progressively enlarging mass. Calcification is seen in up to 20%. Cortical erosion of adjacent bone is a suggestive feature.

How well did you know this?
1
Not at all
2
3
4
5
Perfectly
58
Q

73 A 30 year old undergoes a shoulder x-ray following a fall. No fracture is identified. However, note is made of acromioclavicular joint erosion. Which of the following is least likely?

(a) Lymphoma

(b) Hyperparathyroidism

(c) Rheumatoid arthritis

(d) Scleroderma

(e) Fibrous dysplasia

A

(e) Fibrous dysplasia

Other causes include gout, myeloma, osteomyelitis and previous trauma.

How well did you know this?
1
Not at all
2
3
4
5
Perfectly
59
Q

75 A 13 year old boy presents with a long history of a dull ache_ in his hip. Plain radiographs show a 4cm eccentric lytic lesion of the proximal femoral epiphysis extending to involve the proximal metaphysis. It has a lobulated, well-defined sclerotic margin and areas of calcification within it. There is significant surrounding periosteal reaction. Which of the following is the likeliest diagnosis?

(a) Enchondroma

(b) Chondromyxoid fibroma

(c) Chondrosarcoma

(d) Chondroblastoma

(e) Giant cell tumour

A

(d) Chondroblastoma

The main differential diagnosis for epiphyseal lesions would be GCT. However, these typically arise in a slightly older age group and only cross the physis once it has fused. Chondrosarcomas occur in an older age group (median age 45 years) and chondromyxoid fibromas are rare (usually but not exclusively metaphyseal) cartilaginous tumours arising from the cortex.

How well did you know this?
1
Not at all
2
3
4
5
Perfectly
60
Q
  1. A 17 year old girl presents with pain in the distal forearm which has worsened over the last six to eight weeks. Plain films show an eccentric lytic radiolucency in the distal radius with a soap-bubble appearance. The most likely pathology is:

a. Enchondroma

b. Aneurysmal bone cyst

c. Simple bone cyst

d. Fibrous dysplasia

e. Chondroblastoma

A
  1. b. Aneurysmal bone cyst

Aneurysmal bone cyst is most common in females and 75% occur under 20 years of age. The classic presentation is of pain of relatively acute onset with a rapid increase in severity over 6–12 weeks. Common locations include the spine, with a slight preponderance for the posterior elements, and the metaphysis of long bones – femur, tibia, humerus and fibula. The lesion is usually expansile with thin internal trabeculations giving it the characteristic soap-bubble appearance.

How well did you know this?
1
Not at all
2
3
4
5
Perfectly
61
Q
  1. A 20 year old man presents with an increasingly painful right thigh which is worse at night. Plain films of the area show a lucent area measuring approximately 8–9mm in the distal femur surrounded by extensive sclerosis. The most likely diagnosis is:

a. Osteoblastoma

b. Giant cell tumour

c. Brodie’s abscess

d. Osteoid osteoma

e. Chondroblastoma

A
  1. d. Osteoid osteoma

This most commonly presents in the second and third decades. The male:female ratio is 2.5:1. Classically it presents with increasing pain which is worse at night and often relieved with aspirin. Spinal lesions often lead to painful scoliosis. Almost any site in the body may be affected but the most common regions are the lower limb and spine.

How well did you know this?
1
Not at all
2
3
4
5
Perfectly
62
Q

(ped) 9. A ten year old boy presents with severe localised pain in the distal femur with an associated swelling. Blood films show a leucocytosis and anaemia. At the time of diagnosis he has both lung and bone metastases. The most likely diagnosis is:

a. Osteosarcoma

b. Giant cell tumour

c. Lymphoma

d. Ewing’s sarcoma

e. Clear cell sarcoma

A
  1. d. Ewing’s sarcoma
    `
    Ninety-five per cent present between 4 and 25 years of age. Sixty per cent occur in the long bones, mainly in the metadiaphysis and have a typical moth-eaten destructive appearance on plain film. Metastases to the lung, bone and regional lymph nodes are present in 11–30% of cases at the time of diagnosis.
How well did you know this?
1
Not at all
2
3
4
5
Perfectly
63
Q
  1. A 22 year old man presents to his GP with pain in his right knee which is gradually worsening in severity and is relatively resistant to analgesia. MRI of the knee demonstrates an area of geographic bone destruction in the distal femur with a wide zone of transition. There is marked aneurysmal dilatation of the bone and a fluid-fluid level is present within the lesion. The most likely diagnosis is:

a. Plasmacytoma

b. Simple bone cyst

c. Giant cell tumour

d. Telangiectatic osteosarcoma

e. Parosteal osteosarcoma

A
  1. d. Telangiectatic osteosarcoma

With the MRI finding described, the most likely explanation is that the lesion is a telangiectatic osteosarcoma. This is a rare type of osteosarcoma with a mean age at presentation of 20 years. The most common site is around the knee (62%). Fluid-fluid levels are also seen in giant cell tumours and aneurysmal bone cysts.

How well did you know this?
1
Not at all
2
3
4
5
Perfectly
64
Q
  1. A 74 year old woman with back pain presents to her GP. Initial plain radiographs of her spine show multiple sclerotic metastatic lesions. The most likely primary tumour would be:

a. Renal cell carcinoma

b. Melanoma

c. Bronchial carcinoid

d. Bladder

e. Colorectal carcinoma

A
  1. c. Bronchial carcinoid

The most likely from the above list is a bronchial carcinoid. In men the most likely cause would be prostate. All other conditions are likely to produce lytic metastases than sclerotic.

How well did you know this?
1
Not at all
2
3
4
5
Perfectly
65
Q
  1. An 18 year old student who fell down two stairs and landed on her left knee attends A&E complaining of generalised knee pain but is able to weight bear. No acute bony injury is demonstrated on plain film, however a pedunculated lesion arising from the femoral metaphysis and extending away from the knee joint is seen. The lesion shows continuity with both the marrow and the cortex. The most likely diagnosis is:

a. Osteochondroma

b. Osteoblastoma

c. Osteoid osteoma

d. Chondroblastoma

e. Chondromyxoid fibroma

A
  1. a. Osteochondroma

The description is classic for an osteochondroma or osteocartilagenous exostosis. These lesions are the most common benign growths of the skeleton, are usually found incidentally and are usually asymptomatic unless complications arise. Complications include fracture, vascular compromise, bursa formation and malignant transformation into chondrosarcoma.

How well did you know this?
1
Not at all
2
3
4
5
Perfectly
66
Q
  1. A 25 year old woman attends A&E after falling onto her right hand. A plain film of her hand is taken in order to exclude fracture. No bony injury is seen. On examination, however, there is painless swelling of the right index finger which she says has been present for a few weeks. Incidental note is made of a small central lesion within the medullary cavity of the middle phalanx of the index finger. There is no cortical breakthrough or periosteal reaction but there is bulbous expansion of the bone with thinning of the cortex. The lesion contains dystrophic calcifications. This is most likely to represent:

a. Giant cell tumour of the tendon sheath

b. Unicameral bone cyst

c. Brown tumour

d. Enchondroma

e. Epidermal inclusion cyst

A
  1. d. Enchondroma

This lesion is most likely to be an enchondroma. This is a benign cartilaginous growth in the medullary cavity and is usually asymptomatic. It most commonly occurs in the small bones of the hands and wrist but may also occur in the proximal humerus and proximal femur. Epidermoid inclusion cysts are usually in the distal phalangeal tuft and there is often a history of trauma. A bone cyst would be unusual in the phalanges.

How well did you know this?
1
Not at all
2
3
4
5
Perfectly
67
Q

(Ped) 39. A 12 year old boy attends A&E after falling off his bike. He complains of pain in the right hand. Plain radiographs show no bony injury. However, a small well-rounded lesion is seen in the proximal phalanx of the index finger. This has a ground glass appearance and contains dystrophic calcifications. There is no cortical breakthrough or periosteal reaction. The most likely cause of this lesion is:

a. Simple bone cyst

b. Aneurysmal bone cyst

c. Enchondroma

d. Epidermoid inclusion cyst

e. Fibrous dysplasia

A
  1. c. Enchondroma

The most common location of these benign lesions is the tubular bones of the wrist and hands, and they are usually asymptomatic and therefore diagnosed incidentally. Of the other lesions mentioned, ABC, simple bone cyst and fibrous dysplasia are rare in the hands. An epidermoid inclusion cyst is most likely to occur at the distal phalanx.

How well did you know this?
1
Not at all
2
3
4
5
Perfectly
68
Q
  1. A 20 year old student complains of a six-week history of pain and tenderness in his right thigh associated with a soft-tissue mass. There is no definite history of trauma. CT of the region shows a mass in the right distal femur with well-defined mineralization at the periphery and a less distinct lucent centre. On plain film, there is faint calcification within the lesion and a radiolucent zone separating the lesion from bone. The most likely cause is:

a. Tumoural calcinosis

b. Osteomyelitis

c. Myositis ossificans

d. Parosteal sarcoma

e. Osteosarcoma

A
  1. c. Myositis ossificans

Often there is no distinct history of trauma, although this is the most common cause. It usually occurs in the large muscles of the extremities and in the early stages it can be difficult to distinguish from soft-tissue sarcomas. It is, however, separate from bone, unlike parosteal sarcoma and post-traumatic periostitis. This is a self-limiting condition, most commonly occurring in young athletic adults, with resorption occurring in approximately one year.

How well did you know this?
1
Not at all
2
3
4
5
Perfectly
69
Q
  1. A 60 year old man is referred for an MRI of his left upper leg after noticing a slowly enlarging firm mass measuring approximately 7–8 cm in maximum diameter. The mass is located in the quadriceps muscle group and is causing cortical erosion of adjacent bone. There are poorly defined calcifications within it and MR shows a poorly defined lesion which is isointense to muscle on T1-weighted imaging and hyperintense on T2-weighted imaging. The most likely diagnosis is:

a. Malignant fibrous histiocytoma

b. Benign fibrous histiocytoma

c. Liposarcoma

d. Fibrosarcoma

e. Elastofibroma

A
  1. a. Malignant fibrous histiocytoma

Soft-tissue malignant fibrous histiocytoma is the most common primary malignant softtissue tumour of later adulthood. It is most commonly seen in the lower extremities. It has a metastatic rate of 42% and most commonly metastasises to the lung. Osseous malignant fibrous histiocytoma presents as a painful, tender, rapidly enlarging mass and most commonly arises in the metaphysis of long bones.

How well did you know this?
1
Not at all
2
3
4
5
Perfectly
70
Q
  1. An 18 year old man undergoes a Tc MDP bone scan to investigate pain in the right hip. A ‘hot’ lesion is seen in the right proximal femur. No other lesions are seen. Which of the following lesions would appear as ‘hot’ on a Tc MDP bone scan?

a. Osteopoikilosis

b. Fibrous cortical defect

c. Acute fracture within 12 hours of injury

d. Fibrous dysplasia

e. Haemangioma

A
  1. d. Fibrous dysplasia

The most common site of monostotic fibrous dysplasia is the ribs, followed by proximal femur and craniofacial bones. Three-quarters of cases present before age 30. Other benign lesions causing a ‘hot’ on bone scan include Paget’s disease, brown tumours, aneurysmal bone cysts, osteoid osteoma and chondroblastoma. Acute fractures are not usually ‘hot’ until after the first 24–48 hours.

How well did you know this?
1
Not at all
2
3
4
5
Perfectly
71
Q
  1. A 29 year old woman presents with a painful right knee which has been worsening over the previous few weeks. A plain film of the right knee shows an oval expansile lesion with a radiolucent centre in the metaphyseal region of the proximal tibia. There is a sclerotic margin and geographic bone destruction. There are internal septations and stippled calcification. There is no periosteal reaction. The most likely diagnosis is:

a. Non-ossifying fibroma

b. Chondroblastoma

c. Giant cell tumour

d. Chondromyxoid fibroma

e. Chondrosarcoma

A
  1. d. Chondromyxoid fibroma

This is most commonly seen in the second and third decades and the most common site is the long bones, most often the proximal tibia and distal femur. Non-ossifying fibroma is usually asymptomatic. The appearances of a chondroblastoma would be similar but this would most likely be epiphyseal in location and usually presents in a slightly younger age group.

How well did you know this?
1
Not at all
2
3
4
5
Perfectly
72
Q
  1. A 53 year old woman attends A&E with a short history of dull right heel pain. She is otherwise fit and well and there is no history of trauma. Plain radiographs of the right foot and ankle reveal a 2 cm expansile non-aggressive lesion in the calcaneum. It has a thin, well-defined sclerotic border. There is no periosteal reaction but there is a small calcified central nidus. The most likely cause of the lesion is:

a. Aneurysmal bone cyst

b. Intra-osseous lipoma

c. Lipoblastoma

d. Fibrous dysplasia

e. Desmoplastic fibroma

A
  1. b. Intra-osseous lipoma

The calcaneum is a common location for an intra-osseous lipoma. They do, however, also occur in the extremities, skull and mandible. There is no periosteal reaction unless there is an associated fracture. Imaging features would be similar to those of a unicameral bone cyst. They are often asymptomatic but can present with localised bone pain.

How well did you know this?
1
Not at all
2
3
4
5
Perfectly
73
Q

QUESTION 4
A previously well 80-year-old woman sustains a subcapital fracture of the right neck of femur following a fall onto hard ground. The plain film reveals multiple lytic lesions within the pelvic bones and proximal femora, which are highly suspicious for bone metastases. What is the most likely occult primary lesion?

A Carcinoma of the bladder

B Carcinoma of the breast

C Carcinoma of the bronchus

D Carcinoma of the colon

E Carcinoma of the stomach

A

B Carcinoma of the breast

How well did you know this?
1
Not at all
2
3
4
5
Perfectly
74
Q

A 43-year-old man is investigated for pain related to his left arm. Plain radiography demonstrates a well-defined, lytic lesion in the proximal humerus, with chondroid matrix mineralisation and a narrow zone of transition. There is deep endosteal cortical scalloping and the suggestion of bone expansion. What is the most likely diagnosis?

A Chondroblastoma

B Chondroma

C Chondromyxofibroma

D Chondrosarcoma

E Osteochondroma

A

D Chondrosarcoma

How well did you know this?
1
Not at all
2
3
4
5
Perfectly
75
Q

QUESTION 8
A 45-year-old man unde1went chemotherapy and limb-sparing surgery to treat a soft tissue sarcoma in his left leg. He remains under regular MRI surveillance to detect signs of recurrent disease. What features on MRI would be most suggestive of disease recurrence?

A New areas of high signal on proton density images

B New areas of high signal on Tlw images

C New areas of high signal on T2w images

D New areas of reduced signal on Tl w images

E New areas of reduced signal on T2w images

A

C New areas of high signal on T2w images

How well did you know this?
1
Not at all
2
3
4
5
Perfectly
76
Q

QUESTION 9
An aggressive, lytic bone lesion is observed to destroy the cortex of the second metacarpal in a 70-ycar-old woman. What is the most likely cause for these appearances?

A Metastatic breast cancer

B Metastatic colon cancer

C Metastatic lung cancer

D Metastatic renal cancer

E Metastatic thyroid cancer

A

C Metastatic lung cancer

Skeletal metastases tend to involve the axial rather than appendicular skeleton, reflecting their predilection for bones containing red marrow. Metastases from bronchial carcinoma account for half of all peripheral metastases.

How well did you know this?
1
Not at all
2
3
4
5
Perfectly
77
Q

QUESTION 11
A solitary, lytic lesion with aggressive features is an unexpected incidental finding on radiography of the left knee. Which radiological feature would favour a diagnosis of meastasis rather than primary bone tumour?

A Bone expansion

B Diaphyseal location

C Florid periosteal reaction

D Tumour bone formation

E Soft tissue mass

A

B Diaphyseal location

How well did you know this?
1
Not at all
2
3
4
5
Perfectly
78
Q

QUESTION 18
A 27-year-old man is referred by his GP with progressively painful swelling of his left knee following a minor football injury some weeks ago. The radiograph shows a 5-cm ill-defined lytic lesion within the left distal femoral metaphysis, with a permeative pattern of bone loss and areas of cloud-like ossification. There is an extensive periosteal reaction, predominantly orientated perpendicular to the cortex. What is the most likely diagnosis?

A Aneurysmal bone cyst

B Chondrosarcoma

C Ewing’s sarcoma

D Metastasis

E Osteosarcoma

A

E Osteosarcoma

The appearances are highly aggressive, and characteristic of osteosarcoma. The tumour matrix indicates a tumour of osseous rather than cartilaginous origin, malcing chondrosarcoma highly unlikely.

How well did you know this?
1
Not at all
2
3
4
5
Perfectly
79
Q

(Ped) QUESTION 19
A 5-year-old presents with a 2-month history of a painful right knee. On radiograph, there is a poorly defined lytic lesion in the distal femoral metaphysis, with an associated sunburst type periosteal reaction. An MRI is performed to classify this lesion further. Which one of the following imaging protocols would you choose?

A CT chest, abdomen and pelvis, with MRI Tlw and T2w axial and coronal sequences of the femur

B CT chest, with MRI Tlw and T2w axial, sagittal, coronal sequences of the femur

C Isotope bone scan, CT abdomen with MRI Tlw and T2w sagittal, coronal, axial sequences of the femur

D Isotope bone scan, CT chest with MRI Tlw and T2w sagittal, axial, coronal sequences of the femur

E MRI Tlw and T2w axial, sagittal and coronal sequences of the femur

A

D Isotope bone scan, CT chest with MRI Tlw and T2w sagittal, axial, coronal sequences of the femur

The typical imaging algorithm for children suspected of a primary bone malignancy includes plain radiograph, MRI to cover the entire bone involved by the lesion, bone scintigraphy to look for skeletal metastases and a CT chest for pulmonary metastases.

How well did you know this?
1
Not at all
2
3
4
5
Perfectly
80
Q

(Ped) QUESTION 28
A 13-year-old girl presents with a 3-month history of nonspecific neck ache with muscle spasm. A radiograph of her cervical spine reveals a 3-cm lucent lesion with a sclerotic rim in the posterior elements of C4. There is minimal periosteal reaction. Bone scintigraphy demonstrates a region of intense uptake correlating with the radiographic findings. On MRI, the lesion yields low signal on Tlw and medium to high signal on T2w sequences, as well as mild bone marrow oedema. Which one of the following is the most likely diagnosis?

A Aneurysmal bone cyst (ABC)

B Nonossifying fibroma (NOF)

C Osteoid osteoma

D Osteoblastoma

E Osteosarcoma

A

D Osteoblastoma

The differential diagnoses for a lesion within the posterior elements of a vertebral body include osteoid osteoma, osteoblastoma and an aneurysmal bone cyst.

How well did you know this?
1
Not at all
2
3
4
5
Perfectly
81
Q

QUESTION 33 A 13-year-old boy is referred to the orthopaedic surgeons with a short history of pain and swelling around his left elbow. The radiograph reveals a 4-cm area of permeative bone destruction within the distal diaphysis of the left humerus, with a wide zone of transition. There is an extensive associated soft tissue component and evidence of a ‘hair-on-end’ pattern of periosteal reaction. What is most likely diagnosis?

A Askin tumour

B Chondroblastoma

C Chondromyxoid fibroma

D Ewing’s sarcoma

E Malignant fibrous histiocytoma

A

D Ewing’s sarcoma

Infection and Langerhans cell histiocytosis (LCH) should be considered in the differential diagnosis of a permeative bone lesion in a child. Askin tumour is a rare primitive neuroectodermal rumour of the chest wall in children.

How well did you know this?
1
Not at all
2
3
4
5
Perfectly
82
Q

QUESTION 34 A 2-cm, well-defined lytic bone lesion in the proximal tibial metaphysis is an incidental finding in a 25-year-old woman. The lesion has a thic sclerotic margin and there is a ground glass appearance to the matrix.There is a history of endocrine disturbance and several cafe-au-lait spots are evident on examination. Skeletal scintigraphy subsequently reveals multiple areas of increased activity within the skeleton. What is the most likely diagnosis?

A Gardner’s syndrome

B Mazabraud’s syndrome

C Maffucci’s syndrome

D McCune-Albright

E Ollier’s syndrome

A

D McCune-Albright

McCune-Albright syndrome is characterised by polyostotic fibrous dysplasia, cafe-au-lait spots and endocrine disturbance (most commonly precocious puberty in girls). The rare association of polyostotic fibrous dysplasia and soft tissue mxyomas is Mazabraud’s syndrome.

How well did you know this?
1
Not at all
2
3
4
5
Perfectly
83
Q

QUESTION 36 An 80-year-old man undergoes skeletal scintigraphy for multifocal skeletal pain, malaise and weight loss. The scintigram shows diffusely increased activity throughout the skeleton, with absent renal activity. What is the most likely diagnosis?

A Metastatic bladder cancer

B Metastatic colon cancer

C Metastatic gastric cancer

D Metastatic lung cancer

E Metastatic prostate cancer

A

E Metastatic prostate cancer

How well did you know this?
1
Not at all
2
3
4
5
Perfectly
84
Q

QUESTION 38 A young patient is newly diagnosed with diaphyseal aclasis. What would be the expected imaging findings?

A Multiple enchondromas

B Multiple enostoses

C Multiple osteochondromas

D Multiple osteomas

E Multiple osteoid osteomas

A

C Multiple osteochondromas

How well did you know this?
1
Not at all
2
3
4
5
Perfectly
85
Q

QUESTION 41
A 15-year-old boy is noted to have a solitary lytic lesion expanding the cortex of the proximal tibia. An MRI demonstrates multiple fluid levels. What is the most likely diagnosis?

A Aneurysmal bone cyst

B Enchondroma

C Giant cell tumour

D Osteoblastoma

E Simple bone cyst

A

A Aneurysmal bone cyst

How well did you know this?
1
Not at all
2
3
4
5
Perfectly
86
Q

QUESTION 55
An incidental finding on plain film is a 2-cm lucency within the diaphysis of the right humerus, which exhibits chondroid calcification. Which clinical or radiological feature would favour a diagnosis of chondrosarcoma rather than enchondroma?

A Age less than 20 years

B Circular, curvilinear or nodular calcific densities

C Periosteal reaction

D Slow growth

E Well-defined round or elliptical margin

A

C Periosteal reaction

Outside the hands and feet, chondrosarcoma is five times more common than enchondroma.

How well did you know this?
1
Not at all
2
3
4
5
Perfectly
87
Q

QUESTION 57
A 20-year-old man complains of a 3-month history of pain from his lefr femur. The pain is of insidious onset and is worse at night. As part of his assessment, a bone scintigram is performed, which shows a corresponding area of abnormaliry in the left femoral shaft, characterised by a focus of very high activiry surrounded by a diffuse area of modestly increased activiry. What is the most likely diagnosis?

A Aneurysmal bone cyst

B Enostosis

C Enchondroma

D Osteoid osteoma

E Osteoma

A

D Osteoid osteoma

How well did you know this?
1
Not at all
2
3
4
5
Perfectly
88
Q

QUESTION 72
An 18-year-old man attends his general practitioner with a painful right knee. His radiograph shows a well-defined, lobular, lytic lesion within the proximal tibial epiphysis, extending into the metaphysis. There is a faintly sclerotic margin and no matrix calcification. What is the most likely diagnosis?

A Chondroblastoma

B Chondromyxoid flbroma

C Enchondroma

D Giant cell tumour

E Osteoid osteoma

A

A Chondroblastoma

How well did you know this?
1
Not at all
2
3
4
5
Perfectly
89
Q

QUESTION 80
A radiograph of the left knee of a 35-year-old man reveals a 3-cm lytic lesion sited eccentrically in the proximal tibia. It has a well-defined non-sclerotic margin, and extends to the tibial articular surface. What is the most likely diagnosis?

A Anemysmal bone cyst

B Chondroblastoma

C Giant cell tumour

D Non-ossifying flbroma

E Osteoid osteoma

A

C Giant cell tumour

How well did you know this?
1
Not at all
2
3
4
5
Perfectly
90
Q

(Ped) 3. A 10-year-old girl presents with pain in the left hip which she first noticed when playing sports at school. Apart from pain on movement and several prominent café-au-lait spots, there is nothing else to find on examination. The patient is apyrexic and her WCC and CRP are normal. A plain film of the pelvis reveals a lucent lesion in the proximal femoral metaphysis. The margins of the lesion are well defined and the metaphysis is expanded with adjacent cortical thinning. The lesion extends as far as, but does not involve, the physis. There is GGO in the centre of the lesion. There is no periosteal reaction. What is the most likely diagnosis?

A. Osteomyelitis.

B. Chondroblastoma.

C. Aneurysmal bone cyst.

D. GCT.

E. Fibrous dysplasia.

A
  1. E. Fibrous dysplasia.

The clue here is the cafe-au-lait spots, which are seen in McCune–Albright syndrome (unilateral polyostotic fibrous dysplasia, precocious puberty, and cafe-au-lait spots).
Osteomyelitis is unlikely because of the normal inflammatory markers, lack of systemic symptoms, and lack of periosteal reaction. Aneurysmal bone cyst and GCTs are possibilities (the former may even complicate fibrous dysplasia). Chondroblastoma is an epiphyseal lesion.

91
Q
  1. A 25-year-old man presents with a 4-month history of increasing dull lower back ache. He is otherwise systemically well. He has no neurological signs. An x-ray of the lumbar spine demonstrates a slight scoliosis, with an enlarged sclerotic left pedicle of L3. A subsequent CT scan shows a 3-cm lucent focus within the left pedicle of L3, which has expanded the bone. There is surrounding sclerosis. What is the most likely underlying diagnosis?

A. Osteoid osteoma.

B. Enostosis.

C. Osteoblastoma.

D. Osteomyelitis.

E. Intracortical haemangioma.

A
  1. C. Osteoblastoma.

Osteoblastoma is similar both clinically and histologically to osteoid osteoma, but there are some differences that aid in distinguishing these two entities. Clinically osteoblastoma is typically less painful than osteoid osteoma and does not respond as well to salicylates. An osteoblastoma in the neural arch of the spine is more likely to cause neurological signs, as these lesions are typically larger and more expansile than osteoid osteoma. The lucent nidus seen in osteoid osteoma is usually less than 1.5–2 cm in size, whereas the nidus in osteoblastoma is usually larger than 2 cm at diagnosis and has less surrounding sclerosis. The nidus may or may not have a calcific focus within, in both these diagnoses. The appearance described in the question is the subgroup of osteoblastoma that has similar features to osteoid osteoma. Other appearances on imaging of osteoblastoma include an expansile lesion with multiple small calcifications and a peripheral sclerotic rim or, more rarely, an aggressive appearance with osseous expansion, bone destruction, infiltrating soft tissue, and intermixed matrix calcification.
An enostosis (or bone island) may be giant (greater than 2 cm), but should be well defined and densely sclerotic. It is possible a bone abscess could cause a lytic lesion with surrounding sclerosis, but the patient is systemically well, making infection less likely. A bone abscess is also unlikely to be as expansile as the lesion described. Intracortical haemangioma is a very rare diagnosis, usually within the cortex of a long bone, such as the tibia. On CT there is a hypoattenuating lesion, with spotty internal calcification or a ‘wire-netting’ appearance.

92
Q
  1. A 35-year-male presents with pain in the thigh. A plain radiograph reveals an eccentric expansile lucent lesion without a sclerotic margin but with a narrow zone of transition in the distal femoral metaphysis and epiphysis, which extends to the joint surface. What is the most likely diagnosis?

A. Osteosarcoma.

B. Giant cell tumour (GCT).

C. Metastasis.

D. Aneurysmal bone cyst.

E. Fibrous dysplasia.

A
  1. B. Giant cell tumour (GCT).

This is the classical description and location of a GCT. They occur age 20–40 years, in the long bones and occasionally the sacrum and pelvis. They are lucent, eccentric, and expansile but do not usually produce sclerosis and produce a periosteal reaction in less than a third of patients. They may have a multiloculated appearance. They originate in the metaphysis but extend to the subchondral surface in the skeletally mature. MRI often reveals fluid–fluid levels and some low signal on T2WI due to haemosiderin or collagen deposition. The major differential diagnosis is an aneurysmal bone cyst (ABC), but this classically has a sclerotic margin and usually occurs under 30 years of age (75% occur before the age of 20 years). Fibrous dysplasia would usually present at a younger age in the metaphysis with extension into the diaphysis; a trabeculated/ground-glass appearance is typical with a thick sclerotic margin and endosteal scalloping. Metastasis would be relatively rare at this age and there is no mention of a primary tumour. Osteosarcoma would have a more aggressive appearance with a wide zone of transition, periosteal reaction, cortical destruction, and soft tissue extension.

93
Q

(PED) 8. A 10-year-old boy presents with pain in his left leg following a minor fall whilst playing football. A plain film shows a mixed lytic/sclerotic lesion in the distal femoral metaphysis extending across the physis to involve the epiphysis. The lesion appears centred in the medulla and is causing cortical destruction. There is periosteal reaction and a Codman’s triangle. No significant soft tissue component is visible. What advice do you give as a radiologist?

A. Arrange urgent skeletal survey and CT brain.

B. Advise treatment with antibiotics and arrange outpatient MRI.

C. Arrange CT guided bone biopsy and MIBG scan.

D. Arrange MRI, CT chest, and isotope bone scan.

E. Perform CT chest, abdomen, and pelvis to look for a primary neoplasm elsewhere.

A
  1. D. Arrange MRI, CT chest, and IBS.

The lesion described is probably an osteosarcoma. Ewing’s sarcoma is high on the differential diagnosis, but is less often found in the metaphysis and usually has a soft-tissue component.
Osteosarcomas commonly arise from the metaphysis (only 2–11% arise from the diaphysis). They are most common in the distal femur followed by the proximal tibia. If osteoid matrix is present on plain film, osteosarcoma is the diagnosis until proven otherwise. The appropriate action would be to refer to a regional orthopaedic centre for discussion at a multidisciplinary meeting with imaging followed by biopsy or surgery as thought appropriate. An MRI is used to evaluate local extent, the CT chest and IBS to search for pulmonary and bony metastases, respectively. Surgery is often performed after repeat imaging following the administration of neoadjuvant chemotherapy, when the oedema surrounding the tumour is less pronounced.
The findings are not particularly suspicious for NAI (option A). It is not safe to assume the appearances are caused by osteomyelitis and in any case more urgent investigation would be required to confirm such a diagnosis. A CT of the area might be useful but involves ionizing radiation, which MRI avoids; the MIBG scan would be appropriate for neuroblastoma, which is unlikely in this age group (90% occur before the age of 5) (option C). A metastasis is very unlikely in this age group.

94
Q
  1. A 16-year-old boy fell playing football and hurt his left knee. He has some difficulty weight-bearing and presents to the A&E department. An x-ray of his left knee is performed. This demonstrates a small joint effusion, but no fracture is seen. An approximately 3-cm diameter, well-defined lucent bony lesion, with a thin sclerotic margin, is identified within the proximal epiphysis of the tibia. No internal calcification is evident on plain x-ray. What is the most likely diagnosis for this abnormality?

A. Chondromyxoid fibroma.

B. Enchondroma.

C. GCT.

D. Chondroblastoma.

E. Chondrosarcoma

A
  1. D. Chondroblastoma.

These are radiolucent lesions that typically occupy the epiphysis of long bones in younger people, usually before skeletal maturity. They tend to be less than 4 cm in size, with approximately threequarters having a sclerotic border and one-third a calcified matrix seen on plain radiographs. Chondromyxoid fibromas are rare benign tumours occurring in predominantly the second and third decades of life. They characteristically have sclerotic margins and appear lobulated or ‘bubbly’. They usually arise in the metaphysis of long bones with occasional diaphyseal extension. GCTs tend to occur in young adulthood following skeletal maturity. Patients usually present with pain. The lesion is purely lytic, typically with well-defined, but non-sclerotic, margins. When present in long bones, the lesions are typically metaphyseal, extending across a fused epiphysis to a subarticular location. Periosteal reaction is atypical, but expansile remodelling, cortical penetration, and soft-tissue extension may be seen.

95
Q
  1. A 35-year-old man sprains his right ankle and attends the A&E department. An x-ray of the right ankle is performed. This does not show any evidence of a fracture, but the lateral view does demonstrate a well-defined radiolucent lesion with a faint sclerotic margin in the mid calcaneus. There is some central calcification within the lesion. What is the most likely diagnosis?

A. Simple bone cyst.

B. Normal variant.

C. Enchondroma.

D. Intraosseus lipoma.

E. Bone infarct.

A
  1. D. Intraosseus lipoma.

This is usually asymptomatic and discovered as an incidental finding in adults between 30 and 60 years. The calcaneus is the most common site for intraosseus lipoma, accounting for approximately 32% of cases. The key radiographic features are as described. The central dystrophic calcification seen in approximately 62% of cases is considered pathognomic. The major differential diagnosis of this lesion is a simple bone cyst, although this would not contain central calcification. Unlike bone cysts elsewhere, it is seen at this site into adulthood. Also within the differential diagnosis is a pseudolesion within the calcaneus, caused by a relative paucity of trabecular bone at the same location. Again, central calcification would not be a feature in this phenomenon.
Enchondroma is typically a well-defined osteolytic lesion with central calcification, but it usually has a predilection for tubular bones and would be exceedingly rare in the calcaneus. The described appearances are not typical for bone infarct.

96
Q
  1. A 25-year-old man presents with a painful knee. A plain film reveals a lucent area with a wide zone of transition in the distal femoral metaphysis. MRI reveals fluid–fluid levels. What is the most likely diagnosis?

A. Aneurysmal bone cyst.

B. GCT.

C. Osteosarcoma.

D. Chondroblastoma.

E. Osteoblastoma.

A
  1. C. Osteosarcoma.

Telangiectatic variety of osteosarcoma shows fluid–fluid levels, as does malignant fibrous histiocytoma or any necrotic bone tumour. Telangiectatic osteosarcoma is highly vascular and contains necrotic tissue and blood, with tumour located only along periphery and septa. MRI will reveal enhancing nodularity in latter locations; this will be absent in ABC or GCT. In addition, the plain film findings include bone expansion and cortical breakthrough.
Unlike the other lesions, osteoblastoma does not demonstrate fluid–fluid levels on MRI; it is more common in the posterior elements of the spine than in the long bones. ABC, GCT, and chondroblastoma have a narrower zone of transition on plain film than telangiectatic osteosarcoma. GCT is subarticular. Chondroblastoma is epiphyseal. Other benign causes of fluid–fluid levels include simple bone cysts and fibrous dysplasia.

97
Q
  1. A 16-year-old boy presents with a slowly enlarging, painful swelling in his left lateral chest wall. A CXR shows an expansile lucent lesion arising from the lateral aspect of the left seventh rib. An MRI scan is performed for further evaluation and this demonstrates a lobulated, thin-walled multiseptated lesion with fluid–fluid levels, the dependent layer of which are hyperintense on T1WI. What is most likely diagnosis?

A. Fibrous dysplasia.

B. Aneursymal bone cyst.

C. Enchondroma.

D. Chondroblastoma.

E. Cystic angiomatosis.

A
  1. B. Aneurysmal bone cyst.

ABC accounts for approximately 5% of primary rib lesions, excluding myeloma. The radiological findings and age described in the question are classical for this lesion. Approximately 75% of patients are <20 years of age. The key findings on MRI are the fluid–fluid levels due to the settling of degraded blood products within the cysts. Fluid–fluid levels may also be a feature of other lesions, including GCT and chondroblastoma, but the thin, well-defined margins of an ABC should help to distinguish it from other lesions, particularly in this young age group. Fibrous dysplasia is the most common benign rib lesion. The radiographic appearances are variable, but may show unilateral fusiform enlargement and deformity with cortical thickening and increased trabeculation of one or more ribs. The matrix may appear lytic, may demonstrate a ground-glass appearance, or rarely be sclerotic. Amorphous or irregular calcifications may be seen within the lesion on CT, and MRI shows low to intermediate signal on T1WI and variable T2WI signal. Enchondromas more typically arise in the anterior cartilaginous portion of the rib. Radiographs reveal a lobulated, well-demarcated osteolytic lesion that demonstrates mild expansion and well-defined, sclerotic margins. There is typically matrix calcification and CT is more sensitive at detecting this when the calcification is subtle. MRI shows T2WI hyperintense foci that appear to coalesce with one another and reflect the high fluid content of hyaline cartilage. Chondroblastoma of a rib is reported in the literature, but would be exceedingly rare and typically occurs in the epiphyses of long bones. Cystic haemangiomatosis is a rare disease of disseminated multifocal haemangiomatous or lymphangiomatous lesions in the skeleton and is usually an incidental asymptomatic finding.

98
Q
  1. A 34-year-old man has a 3-month history of right knee pain. There is a remote history of previous right leg trauma. He has an x-ray of the right knee performed, which demonstrates a densely ossified mass immediately adjacent to the posterior cortex of the distal femur. You determine that the differential diagnosis is between post-traumatic myositis ossificans or a parosteal osteosarcoma. Which of the following features on plain x-ray is likely to be most helpful in distinguishing between these diagnoses?

A. Periosteal reaction in the adjacent bone.

B. Presence of lucent areas in the lesion.

C. Pattern of ossification in the lesion.

D. Size of the lesion.

E. Presence of lucent cleft between the lesion and adjacent bone.

A
  1. C. Pattern of ossification in the lesion.

The pattern of ossification is likely to be the most helpful. In post-traumatic myositis ossificans, the ossification occurs classically first at the periphery, whereas in parosteal osteosarcoma, the ossification is diffuse, but predominantly central. Periosteal reaction is typically absent in both these lesions and both lesions may contain lucent areas on plain radiograph.
A lucent cleft between the mass and the bony cortex, representing periosteum, is characteristic in parosteal osteosarcoma, but frequently is not seen as the tumour envelops bone. A thick lucent zone separating myositis ossificans from an adjacent bony cortex is typical, but may not be seen on plain radiograph if the lesion is immediately juxta-cortical.

99
Q
  1. You are reviewing a plain film of pelvis of a 70-year-old woman with recent hip pain. She has a past medical history of bronchial carcinoid. You notice thick, coarsened trabeculae of the left iliac bone, but in comparison to a previous film there is an area of cortical destruction with ‘ring-and-arc’ calcification. There is no adjacent periosteal reaction. Which of the following is the most significant pathology present?

A. Paget’s disease.

B. Chondrosarcoma.

C. Osteosarcoma.

D. Chondroblastoma.

E. Lung metastasis.

A
  1. B. Chondrosarcoma.

The findings describe development of chondrosarcoma in an area of Paget’s disease. While osteosarcoma is more common than either malignant fibrous histiocytoma or chondrosarcoma in Paget’s disease, the ‘ring-and-arc’ calcification in the vignette indicates chondroid rather than osteoid calcification. Sarcomatous transformation in Paget’s is rare, occurring in approximately 1% of cases, but should be suspected if there is new focal pain or swelling. Such lesions, even osteosarcomas, are usually lucent. Periosteal reaction is often absent due to the rapidity of bone destruction.
Other complications of Paget’s disease include those related to osseous weakening (deformity and fracture), arthritis, neurological entrapment, and both benign and malignant GCT. Chondroblastoma may have internal chondroid calcification (60%) but is a well-defined, benign, lucent lesion with a sclerotic rim occurring in the epiphyses of children and young adults. Bronchial carcinoid metastases are usually purely osteoblastic (i.e. sclerotic, not lucent).

100
Q

(Ped) 62. A 12-year-old boy undergoes a plain film of his left knee after minor trauma playing rugby. The film reveals a lucent, cortically based lesion with a sclerotic rim in the metadiaphysis of the proximal tibia. There is no periosteal reaction or soft-tissue component and the boy denies pain prior to the injury. As the reporting radiologist you advise:

A. plain films of the extremities to look for similar lesions

B. MRI of the left lower leg and CT chest

C. Isotope bone scan

D. CT of left lower leg

E. none of the above.

A
  1. E. None of the above.

The correct answer is to do nothing. This is the classical description of a non-ossifying fibroma (NOF). These and fibrous cortical defects are common in children and adolescents, and are usually a painless, incidental finding. They are eccentric, well-defined, cortically-based lesions with marginal sclerosis. They can be expansile. They are most common in the femur and tibia. Fibrous cortical defects measure less than 2 cm whereas NOFs are larger. They routinely heal by sclerosis and disappear. An IBS can show uptake during healing and a CT scan can show apparent cortical breakthrough. Thus the correct management is to do nothing. If there are atypical features, such as pain or periostitis, then additional investigation is indicated.

101
Q
  1. A 5-year-old boy presents with a history of walking difficulty. On examination he is noted to have an antalgic gait and lower limb length discrepancy, with the right limb being shorter than the left. Plain radiographs of the right leg show lobular ossific masses arising from the distal femoral epiphysis and the talus, which resemble osteochondromas. What is the most likely underlying diagnosis?

A. Dysplasia epiphysealis hemimelica (Trevor disease).

B. Multiple epiphyseal dysplasia.

C. Diaphyseal aclasis.

D. Dyschondrosteosis (Leri–Weil disease).

E. Klippel–Trenaunay–Weber syndrome.

A
  1. A. Dysplasia epihysealis hemimelica (Trevor disease).

This is an uncommon developmental disorder relating to the formation of an osteochondromatype lesion at the epiphyses of usually a single lower extremity. The epiphyses most commonly involved are those on either side of the knee or ankle. Typically it is only the medial or lateral side of the epiphyses affected (medial:lateral 2:1). The disease is usually recognized at a young age because of an antalgic gait, palpable mass, varus or valgus deformity, or limb length discrepancy.

102
Q

(Ped) 67. A 5-year-old boy with bilateral wrist pain undergoes a plain film which reveals several peduncuated bony outgrowths from the metaphyses of both radii, which point away from the adjacent joints. What is the most likely diagnosis?

A. Ollier disease.

B. Maffucci syndrome.

C. Morquio syndrome.

D. Diaphyseal aclasia.

E. Hunter syndrome.

A
  1. D. Diaphyseal aclasia.

The description is classic for multiple hereditary osteochondromas/exostoses, also known as diaphyseal aclasia. Osteochondromas are the result of displaced growth plate cartilage, which causes lateral bone growth from the metaphysis. They typically point away from the epiphysis. There is continuity of the normal marrow, cortex, and periosteum between the exostosis and the host bone. The cartilage cap, which is the source of growth, may have some chondoid matrix, but the appearance is otherwise of a deformed but normal bone. They are normally found in the extremities, with 36% around the knee. Their growth normally ceases at skeletal maturity. Symptoms are related to pressure effects on adjacent neural or vascular structures. Less than 1% of solitary osteochondromas undergo malignant transformation to chondrosarcoma. Findings that should alert to this are destruction of exostosis bone, destruction of matrix in the cartilage cap, irregular or thick (>2 cm in adults, >3 cm in children) cap, or growth of the cap after skeletal maturity. Multiple hereditary osteochondromatosis is an uncommon autosomal dominant condition. Patients present with multiple osteochondromas, which cause short stature. The elbow and wrist joints are often deformed. There is a higher risk of malignant transformation than in solitary osteochondromas, probably 2–5%. Ollier disease is the presence of multiple enchondromas and Mafucci syndrome requires, in addition, multiple soft-tissue haemangiomas. Morquio and Hunter syndromes are mucopolysaccharidoses, with their own musculoskeletal abnormalities, which often make an appearance in exams.

103
Q

(CNS) 8 A seven-year-old was referred from the maxillofacial surgeons for assessment of facial asymmetry. X-rays revealed expansion of the right maxilla and frontal bone with a featureless `ground-glass’ appearance. He was also known to have multiple cafe au laic spots. What other feature occurs commonly with this condition?

a Osteosarcoma

b Precocious puberty

C En plaque meningioma

d Dentigerous cysts

e Brown tumours

A

Answer B: Precocious puberty

The imaging features are typical of fibrous dysplasia. Fibrous dysplasia, multiple cafe an laic spots and endocrine disorder such as precocious puberty and hyperthyroidism are seen in McCune-Albright syndrome.

104
Q

8 A 23-year-old male had a chest radiograph as he was short of breath after falling from his bicycle. The mediastinum and lungs appeared normal but an area of fibrous dysplasia was noted. Which type of fibrous dysplasia is most common?

a McCune-Albright syndrome

b Polyostotic

c Craniofacial

d Monostotic

e Familial

A

8 Answer D: Monostotic

Fibrous dysplasia is a skeletal developmental anomaly of bone forming mesenchyme, which manifests as a defect in osteoblastic differentiation and maturation. Almost any bone can be affected. Monostotic is found in 70-80%. Polyostotic is found in 20-30% and can be associated with endocrine dysfunction and cafe an laic spots in 10%. Craniofacial is rarer (10-25% of monostotic, 50% of polyostotic). The familial form is rare.

105
Q

9 An 11-year-old girl presented with hip pain and an X-ray of her pelvis was taken. A lucent lesion with an area of periosteal reaction was visible. The periosteal reaction had a `hair-on-end’ configuration. What is the likely nature of any underlying lesion?

a Normal finding at this age

b Likely benign tumour

c Likely aggressive processes

d Likely haemangioma

e Non-specific - may be benign or aggressive

A

9 Answer C: Likely aggressive processes

Periosteum is composed of two layers, outer fibrous and inner cellular, which has potential for osteoblastic potential. It is more active in childhood and therefore is visualised earlier in children. Periosteal reaction represents either new bone formation or periosteal elevation. The different types of periosteal reaction are: lamellated, solid, speculated, sunburst or hair on end, Codman’s triangle. Tumours presenting with speculated, sunburst, disorganised periosteal reaction or Codman’s triangle are likely to represent aggressive processes.

106
Q

14 You are asked for some advice by a primary care physician who has been consulted by the parents of an 18-month-old child who recently had a series of radiographs of their leg following a fall. No fracture was visible but there is a 1 cm lesion that has been reported as a non-ossifying fibroma. What advice would you convey to the child’s parents?

a Need for referral due to risk of pathological fracture

b No need for specific treatment, spontaneous resolution is likely

c Should be treated with high-dose aspirin

d Are associated with Von Hippel-Lindau (VHL) syndrome

e 10% risk of malignant transformation

A

14 Answer B: No need for specific treatment, spontaneous resolution is likely

Non-ossifying fibromas are also known as fibroxanthoma, fibrous medullary defect and several other synonyms. They are found in up to 40% of children less than two years old. They are generally painless and found in shafts of long bones, mostly lower limb particularly around the knee. They tend to be eccentric within the metaphysis, mostly in the medulla. Multiple lesions have several associations; Neurofibromatosis, fibrous dysplasia, Jaffe-Campanacci syndrome. They tend to be aligned along the axis of the long bone and measure around 2 cm in length. If they are more than 3.3 cm and occupy more than half of the bone diameter, they should be observed. Mostly, these lesions spontaneously resolve.

107
Q

(Ped) 15 A plain radiograph of the lower leg shows a lytic metaphyseal lesion within the tibia with a thinned cortex and fine internal trabeculations. There is no periosteal reaction. A bone scan is performed, which shows increased uptake around the periphery of the lesion with no uptake centrally. An MRI demonstrates multiple cysts of different signal intensity with a low signal rim and heterogeneous enhancement post-gadolinium. Which of the following is the most likely diagnosis?

a Fibrous dysplasia

b Aneurysmal bone cyst

C Enchondroma

d Bone island

e Chondrosarcoma

A

15 Answer B: Aneurysmal bone cyst

Aneurysmal bone cysts are benign expansile lytic lesions containing thin-walled cystic cavities filled with chronic blood products. They present with pain, increasing in severity over 6-12 weeks. The peak age of presentation is 16 years.

108
Q

36 A 17-year-old male was found to have a 3-cm lytic lesion surrounded by marked sclerosis in the distal diaphysis of his right femur. CT confirmed the presence of a nidus with matrix mineralisation and there was no suspicion of other lesions. What is the most likely diagnosis?

a Osteoblastoma

b Enchondroma

c Giant cell tumour

d Ewing sarcoma

e Fibrous dysplasia

A

36 Answer A: Osteoblastoma

Osteoblastomas are clinically and histologically similar to osteoid osteoma with a nidus as a characteristic feature. Enchondromas often have multiple lytic lesions, but there is no nidus. A giant cell tumour is a trabeculated bone lesion. Fibrous dysplasia would be a possibility but the description is classical for an osteoblastoma and fibrous dysplasia more frequently affects the proximal femur.

109
Q

38 Which childhood tumour is almost exclusively located in the epiphysis?

a Osteosarcoma

b Lymphoma

c Chondroblastoma

d Ewing sarcoma

e Non-ossifying fibroma

A

38 Answer C: Chondroblastoma

Chondroblastomas usually occur from 5-20 years and are almost exclusively epiphyseal. Osteosarcomas and non-ossifying fibromas are usually metaphyseal. Lymphoma and Ewing sarcomas are usually diaphyseal.

110
Q

40 A 40-year-old man presented with lethargy and knee pain. A radiograph showed a diaphyseal lesion with bone destruction. What diagnosis would be most likely?

a Osteoblastoma

b Non-ossifying fibroma

C Lymphoma

d Chondroblastoma

e Aneurysmal bone cyst

A

40 Answer C: Lymphoma

Lymphoma of bone is more commonly non-Hodgkin’s lymphoma and is predominantly diaphyseal. None of the other lesions are classically diaphyseal nor would they explain his constitutional symptoms.

111
Q

42 A 35-year-old lady with a two-month history of pain, localised swelling and reduced range of movement of her right knee was shown to have a lytic subarticular lesion in the epiphysis and metaphysis of her distal femur. No surrounding sclerosis or soft-tissue swelling was seen. A subsequent MRI showed heterogeneous signal on both T1- and T2-weighted images. No fluid/fluid levels were visible. What is the most likely diagnosis?

a Metastatic breast deposit

b Enchondroma

C Aneurysmal bone cyst

d Desmoplastic fibroma

e Giant cell tumour

A

42 Answer E: Giant cell tumour

Giant cell tumours are seen in skeletally mature patients. They are epiphyseal/ metaphyseal, most common in the distal femur and do not cause surrounding sclerosis. Desmoplastic fibromas are osteolytic and metaphyseal but usually have well-defined sclerotic margins. Aneurysmal bone cysts do occur in this age but are metaphyseal and one would expect to see fluid levels within the cystic spaces on MRI. Metastatic breast deposits are generally diaphyseal and rare in this age group. Enchondromas are diaphyseal and have matrix mineralisation.

112
Q

43 A 38-year-old female presented with a large, destructive giant cell tumour in her distal femur. What is the most appropriate management?

a Wide resection and reconstruction with allograft

b Curettage and bone graft

c Embolisation

d Radiotherapy

e Conservative management

f Chemotherapy

A

43 Answer A: Wide resection and reconstruction with allograft

Allograft resection is the treatment of choice for a large giant cell tumour causing destruction. A wide resection is preferred as a marginal resection is associated with a high recurrence rate. Chemotherapy has no role in the management of this tumour.

113
Q

44 A 22-year-old man presented with pain and swelling of his right knee having knocked it while playing football. The X-ray revealed a destructive lesion in the proximal tibia with a mixed sclerotic/lytic appearance. A’hair on end’ type periosteal reaction was also visible. An MRI showed intermediate signal with foci of high signal intensity on Ti- and heterogeneous high-signal T2-weighted images. What is the most likely diagnosis?

a Osteoid oteoma

b Osteochondroma

c Osteosarcoma

d Chondromyxoid fibroma

e Chondrosarcoma

A

44 Answer C: Osteosarcoma

Osteosarcoma is a high-grade intramedullary tumour. It has high signal on T2 W MRI due to replacement of the normal bone marrow and foci of high signal on T1 W MRI because of central haemorrhage. An osteoid osteoma would not have significant periosteal reaction. Osteochondroma is painless with no periosteal reaction. Chondromyxoid fibroma occurs in the third or fourth decade. It is a lucent lesion with a sclerotic rim. Chondrosarcoma is unlikely in this age group, usually present over 40 years of age.

114
Q

45 A 19-year-old girl presented with discomfort in her left leg of gradual onset with no history of trauma. A radiograph showed a translucent area with a thin sclerotic rim in the epiphysis of her left femur. An MRI was subsequently performed to further characterise this lesion and showed a lobulated margin of low signal intensity corresponding to the sclerotic margin on the X-ray. What is the most likely diagnosis?

a Clear cell chondrosarcoma

b Chondroblastoma

C Osteosarcoma

d Fibrosarcoma

e Osteoma

A

45 Answer B: Chondroblastoma

Chondroblastoma typically occur in this age group and are epiphyseal, welldefined lucencies with a thin sclerotic rim. Osteosarcoma, fibrosarcoma and clear cell chondrosarcoma would be expected to have more aggressive features. Osteomas occur in the elderly population and tend to be sclerotic

115
Q

(Ped) 45 A plain radiograph is performed on a four-year-old with progressive deformity of the right hand. This shows multiple radiolucent expansile lesions within the metacarpals and phalanges, some of which contain punctate calcifications. Which of the following is the most likely diagnosis?

a Chondroblastoma

b Chondromyxoid fibroma

c Langerhans cell histiocytosis

d Enchondromatosis

e Aneurysmal bone cyst

A

45 Answer D: Enchondromatosis

Enchondromatosis, or Oilier disease, is a non-hereditary failure of cartilage ossification. It can involve the long bones, causing leg or arm shortening, or the tubular bones of the hands and feet, resulting in deformity.

116
Q

46 A 76-year-old man presented with pain in his left femur. There was a long history of Paget’s disease and bony tenderness about his knee on the medial femoral condyle. He denied any history of trauma and a radiograph showed an area with sunburst periosteal reaction within the Pagetoid bone. What is the most likely diagnosis?

a Osteosarcoma

b Osteochondroma

c Chondrosarcoma

d Fibrous dysplasia

e Osteomalacia

A

46 Answer A: Osteosarcoma

Osteosarcomas are the most common and aggressive primary bone tumours. Despite the potential of arising in any bone, the majority arise in the metaphyseal growth plates of long bones. Other sites are the pelvis and jaw. The peak incidence is in the second and third decades but there is a second peak in the seventh and eighth decades, chiefly as a result of malignant transformation in Paget’s disease.

117
Q

47 A 12 year old was shown to have an eccentric, lobulated lesion involving the cortex and medulla of the proximal tibial metaphysis with a sclerotic endosteal border but no periosteal reaction. A subsequent CT showed no soft-tissue extension or fluid levels. What is the most likely diagnosis?

a Aneurysmal bone cyst

b Enchondroma

c Chondrosarcoma

d Polyostotic fibrous dysplasia

e Chondromyxoid fibroma

A

47 Answer E: Chondromyxoid fibroma

The proximal tibial metaphysis is a classic location for a chondromyxoid fibroma which is always benign, hence there is a lack of periosteal reaction and softtissue involvement. Aneurysmal bone cysts do occur in the metaphysis; usually a thinned cortex and fluid-fluid levels are visible.

118
Q

48 An 11-year-old boy with hip pain was found to have an osteoid osteoma of his right neck of femur. What is the most appropriate treatment of this lesion?

a Chemotherapy

b Resection and prosthetic replacement

C Percutaneous radiofrequency ablation (RFA)

d Low-dose focal irradiation (800 cGy)

e Curettage, cryotherapy and bone grafting

A

48 Answer C: Percutaneous radiofrequency ablation

Percutaneous RFA is the treatment of choice for osteoid osteoma and primary success rates of up to 90% are reported. With secondary recurrence, repeat RFA leads to an even higher success rate approaching 100%. Chemotherapy and irradiation are not indicated in the treatment of these tumours. Resection, curettage, cryotherapy, bone grafting and prosthetic replacement are invasive with a greater risk of incomplete excision.

119
Q

(Ped) 48 A nine-year-old girl is investigated for precocious puberty. On examination she is noted to have two large cafe an laic spots with irregular edges on the right side of her back and to have a mild leg length discrepancy with a shorter right leg. A plain radiograph of the right leg is performed. Which of the following is the most likely finding?

a Several `ground-glass’ medullary lesions within the proximal femur with endosteal scalloping

b Anterolateral bowing of the lower half of the tibia

C Absent fibula

d Pseudoarthrosis of the tibia

e Generalised osteoporosis of the femur, tibia and fibula

A

48 Answer A: Several `ground-glass’ medullary lesions within the proximal femur with endosteal scalloping

The girl has McCune-Albright syndrome, which consists of at least two out of the triad of. polyostotic fibrous dysplasia, cafe an laic spots and endocrinopathy including precocious puberty. The cafe an laic spots in McCune-Albright syndrome tend to be fewer in number and more prominent than the smooth-edged lesions seen in neurofibromatosis. The other radiographic findings listed above are all seen in neurofibromatosis.

120
Q

49 A patient presented with a long history of pain in her wrist and no history of trauma. A radiograph showed a sharply marginated lesion with multiple foci of ‘popcorn’-like calcification. Bone densitometry was within normal limits. An MRI showed hypointensity to muscle on both T1- and T2-weighted images and no fluid collections. What is the most likely diagnosis?

a Fibrous dysplasia

b Osteogenesis imperfecta

C Multiple enchondromas

d Calcifying solitary bone cyst

e Aneurysmal bone cyst

A

49 Answer A: Fibrous dysplasia

The popcorn-shaped calcifications are cartilage-producing nodules, which are typical of fibrous dysplasia.

121
Q

51 A mildly expansile lytic lesion was noted in the femur of an otherwise well 14-year-old boy. This was well defined and unilocular and extended from metaphysis into diaphysis. The remaining bone was well mineralised and there were no associated aggressive features. What is the most likely diagnosis?

a Aneurysmal bone cyst

b Fibrous dysplasia

c Giant cell tumour

d Simple bone cyst

e Brown tumour

A

51 Answer D: Simple bone cyst

Simple bone cysts often have prominent fluid levels. They commonly present in the second decade and are slightly expansile. Aneurysmal bone cysts have more marked expansion with fluid-fluid levels and tend to occur in an older age group. Fibrous dysplasia is unlikely as it is not an entirely lytic lesion. Giant cell tumour is lytic but often has septations. With brown tumours there is osteoclastic activity due to hyperparathyroidism, so there would usually be osteopenia.

122
Q

52 A six-year-old child presented unwell with a history of 12 weeks of pain and swelling over the left knee. On examination they were pyrexial and noted to have a knee effusion. Blood tests showed a raised erythrocyte sedimentation rate (ESR) and a radiograph showed an aggressive destructive lesion with a permeative pattern of bone destruction in the distal femur and a lamellated (‘onion skin’) periosteal reaction. What is the most likely diagnosis?

a Ewing sarcoma

b Brodie’s abscess

C Trauma

d Secondary neuroblastoma

e Eosinophilic granuloma

A

52 Answer A: Ewing sarcoma

With Ewing sarcoma `onion skinning’ reflects the periodic activity of the sarcoma interspersed with quiescent periods. Brodie’s abscess is high on the differential given the systemic upset, but usually has a lytic area with surrounding sclerosis. Trauma is not likely due to the insidious onset. Neuroblastoma typically occurs under the age of five.

123
Q

54 A six-year-old boy was brought to the GP with his parents after sudden onset of pain in the proximal humerus. Radiographs reveal fracture through a simple bone cyst. What imaging features would be most typical?

a Thick internal septa

b Well-defined lucency but no sclerotic rim

C Subarticular epiphyseal location

d Long axis perpendicular to bone

e Thinned cortex with mild expansion

A

54 Answer E: Thinned cortex with mild expansion

Simple bone cysts are fluid-filled lesions of unknown aetiology. They are most common in males in the first two decades of life and are metaphyseal lesions which migrate into the diaphysis over time. They may have thin internal septae and a thin sclerotic rim is often present.

124
Q

55 A 32-year-old man presents with pain and swelling in his right shoulder of four years’ duration that has gradually been getting worse recently. He works as a mechanic and has been having increasing difficulty using his right arm for the last few months but is otherwise well. On examination he has an obviously swollen right shoulder. A firm mass is palpable over the lateral aspect of his proximal humerus, extending distally to his elbow. There are also multiple palpable axillary lymph nodes. A radiograph of his shoulder shows a mottled, permeative lucency in the head of the humerus. What is the most likely diagnosis?

a Eosinophilic granuloma

b Ewing sarcoma

C Metastatic deposit

d Non-Hodgkin’s lymphoma

e Osteomyelitis

A

55 Answer D: Non-Hodgkin’s lymphoma

It is very rare for Hodgkin’s disease to present in bone and most intraosseous lymphomas are of the non-Hodgkin’s type.

125
Q

56 A nine-year-old girl presented with pain and swelling of her left lower leg over the anteromedial aspect of her tibia. A radiograph showed an aggressive lesion with mixed lytic/sclerotic appearances in the proximal tibial metaphysic. A biopsy was taken which showed odd-looking pleomorphic cells and stained positive for osteoid. What is the most likely diagnosis?

a Granulocytic sarcoma

b Ewing sarcoma

C Osteosarcoma

d Aneurysmal bone cyst

e Osteoid osteoma

A

56 Answer C: Osteosarcoma

Osteosarcoma is an aggressive lesion that can be mixed lytic/sclerotic in appearance and is common at this age and site. Granulocytic sarcoma occurs in patients with leukaemia or other myeloproliferative disorder but is rarely sclerotic. Ewing sarcoma has a moth-eaten appearance and is usually lytic. Aneurysmal bone cysts are not aggressive. Osteoid osteoma has a nidus and no bony destruction.

126
Q

(Ped) 56 A 12-year-old boy presents with a six-week history of a painful leg mass. He has a fever and leucocytosis. A plain radiograph of the leg demonstrates a poorly defined lytic lesion in the tibial diaphysis. There is a lamellar periosteal reaction and penetration into the soft tissues with preservation of the tissue planes. Which of the following is the most likely diagnosis?

a Osteomyelitis

b Eosinophilic granuloma

C Neuroblastoma

d Osteosarcoma

e Ewing’s sarcoma

A

56 Answer E: Ewing’s sarcoma

Ewing’s sarcoma is the most common bone tumour in children. The lesions are characteristically illdefined mottled moth eaten' areas of bone destruction with a periosteal reaction that has an onion skin’ appearance. There is usually an associated soft-tissue mass. Osteomyelitis can have similar appearances but usually a shorter history. In eosinophilic granuloma there is a solid periosteal reaction. Osteosarcoma does not cause a lamellar periosteal reaction and frequently has ossification in the soft tissues.

127
Q

57 A five-year-old boy was noted to have multiple hard swellings predominately around his knees, and radiographs showed multiple bony outgrowths extending laterally from the bone and pointingaway from the nearest joint. The overlying cortex remained in continuity with the native bone although there were some modelling deformities associated with the lesions. What are the individual lesions?

a Osteosarcoma

b Osteochondroma

c Chondroblastoma

d Chondrosarcoma

e Ivory osteoma

A

57 Answer B: Osteochondroma

Diaphyseal aclasis is the condition of multiple hereditary osteochondromas. It is an autosomal dominant disorder in which multiple osteochondromas are seen throughout the skeleton, preferentiallyaffecting the long bones. It is associated with short stature and asymmetrical growth leading to deformities.

128
Q

61 Which statement best describes the characteristics of a chondromyxoid fibroma?

a Geographic bone destruction with periosteal reaction

b Poorly defined lesion with no sclerotic rim and septation

c Geographic bone destruction with no sclerotic rim

d Poorly defined lucency with periosteal reaction and expansions

e Geographic bone destruction with a prominent sclerotic rim

A

61 Answer E: Geographic bone destruction with a prominent sclerotic rim

No periosteal reaction is present unless there is a fracture. Internal septation is common and there is often thinning of the overlying cortex

129
Q

(CNS) 61 On plain film X-ray of the lumbar spine a dense single vertebra is seen (ivory vertebra). The vertebra is not expanded and the trabeculae are not thickened. There is no evidence of disc space narrowing, periosteal reaction or soft tissue mass. What is the most likely diagnosis?

a Low-grade infection

b Paget’s disease

C Sclerotic metastasis

d Lymphoma

e Haemangioma

A

61 Answer D: Lymphoma

The term ivory vertebra describes single or multiple very dense vertebrae and all the options listed are causes. Paget’s disease and haemangiomas show coarse reticulation and some expansion. Low grade infections are associated with endplate destruction, disc narrowing and paraspinal masses. Sclerotic metastases generally do not show expansion but a history of underlying malignancy is often available and there may be evidence of metastatic disease elsewhere.

130
Q

62 At what age does a chondroblastoma usually present?

a 0-5 years

b 10-20 years

c 30-40 years

d 50-60-years

e Over 70 years

A

62 Answer B: 10-20 years

Chondroblastomas present as a well-defined lucency usually in the femur, proximal humerus or proximal tibia. Internal calcification is visible in 60% and there is usually florid marrow oedema on MRI.

131
Q

63 How are enchondromas most likely to present?

a Anaemia

b Fever

c Malaise

d Malignant degeneration

e Pathologic fracture

A

63 Answer E: Pathologic fracture

Malignant degeneration is rare, particularly in solitary peripheral lesions. It is more common with multiple lesions and in more central lesions. Unless it occurs no systemic symptoms occur.

132
Q

65 What is the sex distribution of patients who develop Ewing sarcoma?

a 4 Male: 1 Female

b 2 Male: 1 Female

c 1 Male: 1 Female

d 1 Male: 2 Female

e 1 Male: 4 Female

A

65 Answer D: 1 Male: 2 Female

Most bone tumours either have an even sex distribution or are more common in men except Ewing’s, aneurismal bone cysts, haemangiomas and giant cell tumours.

133
Q

66 A 40-year-old female presented feeling unwell and was found to have a raised serum calcium and alkaline phosphatase with a borderline low phosphate. She had several recent episodes of renal colic and felt generally weak. An X-ray of her pelvis showed a lytic lesion that was associated with an underlying endocrine abnormality. What is the likely imaging appearance?

a Poorly defined central lesion with prominent periosteal reaction

b Poorly defined eccentric lesion with periosteal reaction

c Poorly defined central lesion with no periosteal reaction

d Well-defined cortical lesion with no periosteal reaction

e Well-defined central lesion with prominent periosteal reaction

A

66 Answer D: Well-defined cortical lesion with no periosteal reaction

Brown tumours occur in hyperparathyroidism and are most common in the primary form. They are also known as osteoclastomas due to parathyroid hormone-stimulated osteoclastic activity. These tumours are eccentric/cortical and are usually solitary.

134
Q

(CNS) 8 A 54-year-old underwent a CT of the paranasal sinuses for functional endoscopic sinus surgery (FESS) planning. The right frontal sinus appeared significantly larger than the left and was opacified. At the base there was a dense lesion, which appeared to contain compact cortical bone and protruded into the sinus cavity. On MRI the lesion appeared as a signal void. What is the most likely diagnosis?

a Ivory osteoma

b Fibrous dysplasia

C Meningioma

d Ossifying fibroma

e Foreign body

A

8 Answer A: Ivory osteoma

Ivory osteomas are benign hamartomas of the bone, which are relatively common. They may be associated with other conditions such as Gardener’s syndrome. They most commonly occur in the frontal and ethmoid sinuses and may cause obstruction of the sinus ostium. Ivory osteomas are dense areas of compact bone and are well-defined sclerotic bone on CT and low signal on T1-weighted imaging.

135
Q

12 A 17-year-old boy presented to the Emergency Department five weeks after falling off his bike and hitting his thigh. Plain films showed some faint calcifications in the tissue adjacent to the femur and some simple periosteal reaction. A CT scan confirmed peripheral ossification. What is the most likely diagnosis?

a Osteosarcoma

b Fibrosarcoma

C Stress fracture

d Chondrosarcoma

e Myositis ossificans

A

12 Answer E: Myositis ossificans

This is also known as heterotopic ossification, pseudomalignant tumour of soft tissue and extraosseous localised non-neoplastic bone and cartilage formation. It is a benign solitary selflimiting ossifying soft tissue mass typically occurring within skeletal muscle. Direct trauma is the cause in 75%. The pathology is of a lesion surrounded by compressed fibrous connective tissue and surrounded by atrophic skeletal muscle. The symptoms include pain, tenderness and soft-tissue mass. It has a progressive natural history where it resolves spontaneously, which is reflected in its plain film and MR characteristics.

136
Q

(Ped) 23 A nine-year-old girl presents with severe pain in the left leg following a trivial fall. A radiograph shows an abnormality within the proximal femur. There is a lucent intramedullary lesion with a thinned cortex and an apparent fragment of bone within the inferior part of the lesion. Which of the following is the most likely diagnosis?

a Simple bone cyst

b Osteosarcoma

C Langerhans cell histiocytosis

d Aneurysmal bone cyst

e Ewing’s sarcoma

A

23 Answer A: Simple bone cyst

Simple bone cysts or unicameral bone cysts are most frequently found in the metaphyses of long tubular bones, particularly the proximal humerus, femur and tibia. Fracture is a common complication with the fractured piece of cortex falling to the dependent part of the cyst, the `fallen fragment sign’, described above.

137
Q

36 A 17-year-old female noticed pain and swelling around her distal thigh, particularly while exercising. She had no history of trauma. On examination there was localised tenderness above her knee. Radiographically, a lytic expansile lesion in the metaphysis of the distal femur surrounded by a very thin rim of cortex was visible. There was no periosteal reaction. What is the most likely disease process?

a Ewing sarcoma

b Osteosarcoma

c Fibrous dysplasia

d Aneurysmal bone cyst

e Chondrosarcoma

A

36 Answer D: Aneurysmal bone cyst

The absence of periosteal reaction and presence of intact cortex make Ewing sarcoma, osteosarcoma or chondrosarcoma unlikely. Fibrous dysplasia is possible but the appearance and location would be atypical. The major differential is a giant cell tumour.

138
Q

37 A 14-year-old boy with a three-week history of persistent pain in his right leg presented to hospital. He was noted to have a temperature of 37.8°C and a radiograph of his leg showed a diaphyseal mass in the right femur with overlying cortical erosion and soft-tissue swelling. A bone biopsy was undertaken showing numerous small round blue cells. Which of the following tumours is he most likely to have?

a Neuroblastorna

b Medulloblastoma metastasis

c Osteoblastoma

d Ewing’s sarcoma

e Chondroblastoma

A

37 Answer D: Ewing sarcoma

Ewing sarcoma is commonest in this age group and in a diaphyseal location. Neuroblastoma microscopically does contain small blue cells but typically locates in the adrenal gland of children. Medulloblastoma is composed of small blue cells but typically is found in posterior fossa of children. Osteoblastoma is essentially a large osteoid osteoma. Chondroblastomas are epiphyseal tumours.

139
Q

39 An elderly gentleman presented with a two-month history of back pain. There was no history of significant osteoarthritis and no history of trauma. Plain radiographs revealed multiple sclerotic lesions scattered throughout the spine, which were confirmed as sites of increased uptake on a Tc- 99 radionuclide bone scan. What primary tumour is most likely?

a Breast

b Thyroid

c Kidney

d Prostate

e Lung

A

39 Answer D: Prostate

Prostate cancer is well-known for causing osteoblastic metastases. Breast and lung cancer produce mixed lytic and sclerotic metastases. Kidney and thyroid cancers are usually purely lytic.

140
Q

40 A 21-year-old man presented with pain in his knee following a fall and a radiograph demonstrated a bone tumour in his patella. What lesion is most likely?

a Osteosarcoma

b Osteochondroma

c Chondroblastoma

d Non-ossifying fibroma

e Adamantinoma

A

40 Answer C: Chondroblastoma

The patella is considered an epiphyseal equivalent and hence the same differential applies. Osteosarcomas, osteochondromas and non-ossifying fibromas are metaphyseal lesions. Adamantinomas are diaphyseal.

141
Q

41 A 14-year-old female presented with arm pain with no history of trauma and was found to have an eccentric non-expansile lucent lesion in her proximal humerus. There was no soft tissue mass or periosteal reaction and a subsequent MRI showed multiple fluid-fluid levels. What is the most likely diagnosis?

a Giant cell tumour

b Aneurysmal bone cyst

c Enchondroma

d Osteoblastoma

e Fibrous dysplasia

A

41 Answer B: Aneurysmal bone cyst

The appearance of fluid-fluid levels on MRI is characteristic and the appearance given on plain film is typical.

142
Q

42 A 26-year-old woman was found to have a suspected giant cell tumour in her proximal humerus. What is the most appropriate management?

a Biopsy and surgical curettage, no follow-up required

b Biopsy, `extended’ curettage and packing with follow-up X-rays

c Curettage alone

d Biopsy with follow up X-rays

e Conservative management

A

42 Answer B: Biopsy, `extended’ curettage and packing with follow-up X-rays

A biopsy is usually performed to confirm the diagnosis and exclude malignancy. Extended curettage is then performed to reduce the risk of recurrence (approx 30%).

143
Q

43 A 30-year-old man was found to have monostotic fibrous dysplasia affecting his humerus. Which part of the bone is more likely to be involved?

a Epiphyseal

b Epiphyseal and metaphyseal

c Metaphyseal

d Metaphyseal and diaphyseal

e Diaphyseal

A

43 Answer E: Diaphyseal

144
Q

44 A 22-year-old man was found to have an osteosarcoma of his proximal tibia. How should this patient be managed?

a Curettage and bone grafting

b Above knee amputation

c Wide resection only

d Wide resection and chemotherapy

e Wide resection and radiotherapy

A

44 Answer D: Wide resection and chemotherapy

Wide surgical margins and limb-sparing resection combined with chemotherapy is the most successful regime. Pre- and post-operative chemotherapy significantly improves outcome.

145
Q

45 A 55-year-old man presented with leg pain and a suspicious lesion was seen on a radiograph. Bone biopsy showed an osteogenic sarcoma and referral to the regional oncology centre was undertaken. What is the most important initial staging investigation?

a Chest X-ray

b MRI brain

c Lymphoscintigram

d US liver

e NU renal tract

A

45 Answer A: Chest X-ray

Lung metastases are the most common metastases from sarcomas, including osteogenic sarcomas. He may also need a CT but of the available options a CXR is the best initial test.

146
Q

46 A 68-year-old man presents with insidious onset right-sided shoulder pain. He gives no history of trauma but has recently lost some weight. On examination, he has a markedly reduced range of movement of the shoulder with a palpable mass. An AP radiograph shows `popcorn’ calcification in the head of the humerus. What is the most likely diagnosis?

a Chondroblastoma

b Osteosarcoma

C Chondrosarcoma

d Multiple myelorna

e Desmoplastic fibroma

A

46 Answer C: Chondrosarcoma

Chondrosarcoma generally occurs in adults over the age of 60 and the proximal humerus is a common location. `Popcorn’ calcification is a typical radiological sign. Chondroblastoma occurs in a much younger patient population (10-30 years) and typically has a sclerotic border.

147
Q

47 A 27-year-old female with a long-standing history of back pain was found to have a scoliosis. A plain radiograph demonstrated the absence of the right pedicle of T9 and a bone scan showed a hot spot at this site. CT confirmed a lytic expansile lesion with an associated soft-tissue mass. Biopsy showed the lesion to be benign and as the patient declined treatment, follow-up showed slow growth. What is the most likely diagnosis?

a Aneurysmal bone cyst

b Osteoblastoma

C Osteoid osteoma

d Chondromyxoid fibroma

e Langerhans cell histiocytosis

A

47 Answer B: Osteoblastoma

Osteoblastoma, in contrast to many other benign lesions, are frequently associated with an extracortical mass.

148
Q

48 A 12-year-old boy presented with pain in his thigh for a period of three weeks, which is worse at night and relieved by aspirin. Radiographs showed a small lucent area in the proximal femoral cortex surrounded by sclerosis. What is the most likely diagnosis?

a Osteochondroma

b Eosinophilic granuloma

C Multiple myeloma

d Enchondroma

e Osteoid osteoma

A

48 Answer E: Osteoid osteoma

This is a typical description of an osteoid osteoma with a central nidus.

149
Q

49 A nine-year-old girl with past history of cyclical vaginal bleeding presented with an eight-month history of hip and ankle pain. Multiple hyperpigmented skin lesions (cafe an lait spots) were seen on examination. Plain radiographs showed several `ground-glass’ lesions in the proximal femur and distal tibia. What is likely diagnosis?

a Paget’s disease

b Hand-Schuller-Christian disease

C Gardner’s syndrome

d Letterer-Siwe disease

e McCune-Albright syndrome

A

49 Answer E: McCune-Albright syndrome

McCune-Albright syndrome is the triad of fibrous dysplasia, cafe an laic spots and endocrine dysfunction. Bone lesions in fibrous dysplasia typically have a’ground- glass’ appearance. Hand- Schuller-Christian disease is the triad of exophthalmos, diabetes insipidus and lytic skull lesions. In Gardner’s syndrome there are multiple neoplasms of bone and intestinal polyps. Letterer-Siwe disease is a severe form of histiocytosis X.

150
Q

50 A 25-year-old man presented with a painful leg following a fall playing football. A radiograph showed a pathological fracture of his femur through a well-defined, lytic lesion in the medulla, which was mildly expansile and with a ground-glass matrix. Other than at the fracture there was no disruption of the cortex or soft-tissue mass. What is the most likely diagnosis?

a Fibrous dysplasia

b Metastasis

C Multiple myeloma

d Lymphoma

e Maffucci’s syndrome

A

50 Answer A: Fibrous dysplasia

Fibrous dysplasia typically has a ground-glass appearance, a thick sclerotic border and often presents as a result of a pathological fracture.

151
Q

51 An eight-year-old boy presented with an insidious onset of right-sided shoulder pain. A radiograph showed an aggressive-looking lytic lesion in the metadiaphysis of the proximal humerus. An MRI was subsequently performed, which showed a circumferential extraosseous extension of the tumour with intermediate signal (similar to fat) on T2 -weighted images with surrounding oedema. What is the most likely diagnosis?

a Osteomyelitis

b Lymphoma

C Ewing sarcoma

d Acute bone infarction

e Chondroblastoma

A

51 Answer C: Ewing sarcoma

The intermediate signal on T2 is due to the presence of hypercellular tumour combined, which suggests extensive soft-tissue disease, a characteristic feature of Ewing sarcoma. Osteomyelitis can mimic changes of an aggressive bone tumour but is not the most likely diagnosis here. Lymphoma tends to affect older children. Chondroblastoma does occur in this age group but arises in the epiphysis and has a sclerotic rim.

152
Q

52 An X-ray of a 32-year-old gentleman shows a subarticular multiloculated, lytic lesion in the wrist with minimal expansile modelling centred in the metaepiphysis. What is the most likely diagnosis?

a Aneurysmal bone cyst

b Enchondroma

C Intraosseous ganglion

d Unicameral bone cyst

e Giant cell tumour

A

52 Answer E: Giant cell tumour

These are typical findings of a GCT and the age of the patient is also typical. Aneurysmal bone cysts tend to cause more expansion and to be more diaphyseal. Unicameral bone cyst (also known as simple bone cyst), can be mildly expansile but not multiloculated.

153
Q

56 A 28-year-old gentleman was known to have a lytic lesion in the metaphysis of his proximal tibia and underwent an MRI scan for further assessment. This showed a well-defined lesion with low signal intensity on both T1- and T2-weighted scans. What is the most likely diagnosis?

a Intraosseus ganglion

b Clear cell chondrosarcoma

c Giant cell tumour

d Solitary subchondral cyst

e Brodie’s abscess

A

56 Answer C: Giant cell tumour

Giant cell tumours are metaphyseal lesions that extend to the articular surface. They are most common in the third and fourth decades of life. The low signal intensity on Ti- and T2-weighted images seen with giant cell tumours is due to haemosiderin deposition. All the other diagnoses have high signal intensity on T2 -weighted images.

154
Q

53 A 15-year-old boy was recently diagnosed with a Ewing sarcoma of his vertebral column. Which segment is most likely to be affected?

a Cervical spine

b Thoracic spine

C Lumbar spine

d Sacrum

e Coccyx

A

53 Answer D: Sacrum

When Ewing’s affects the spine the sacrum is most commonly affected, followed by the lumbar, thoracic and cervical regions in that order.

155
Q

54 A middle-aged female presented with insidious onset of pain in her left ring finger and no history of trauma. A radiograph showed a solitary wellcircumscribed, round lytic lesion with cortical expansion in the diaphysis of the middle phalanx. There is no associated soft-tissue mass. What is the most likely diagnosis?

a Enchondroma

b Chondrosarcoma

C Medullary bone infarction

d Osteoid osteoma

e Eosinophilic granuloma

A

54 Answer A: Enchondroma

The description is typical. The risk of malignant degeneration in solitary peripheral enchondromas is very small.

156
Q

55 A 19-year-old man presented with a history of knee pain and swelling aggravated by movement. His blood tests showed a mildly raised white cell count and raised alkaline phosphatase (ALP). A radiograph showed a Codman’s triangle at the margin of a soft-tissue mass. What is the most likely diagnosis?

a Brodie’s abscess

b Osteosarcoma

C Ewing sarcoma

d Haematoma

e Histiocytosis X

A

55 Answer B: Osteosarcoma

The most common site for osteosarcoma is the distal femur. Pain is worse with activity and with a high ALP the patient is more likely to have lung metastases.

157
Q

58 A nine-year-old girl presented with left hip and knee pain. A subtle welldefined radiolucency was visible in the basi-cervical region of the left femur measuring 6 mm and within this lesion there was a smaller dense opacity. Extensive cortical sclerosis was present around the lesion. What is the most likely diagnosis?

a Osteoblastoma

b Chronic sclerosing osteomyelitis

C Foreign body granuloma

d Subperiosteal haematoma

e Osteoid osteoma

A

58 Answer E: Osteoid osteoma

The appearance of a central nidus with surrounding sclerosis is common. Osteoblastoma tends to have irregular margins and is larger (2-10 cm). Chronic sclerosing osteomyelitis is also usually irregular with more bone destruction and periosteal reaction although it is possible to mistake a sequestrum for the nidus.

158
Q

(CNS) 58 A 16-year-old female was investigated for a lucent lesion in her spine and a primary bone tumour was suspected. As part of the work-up a three-phase bone scan was performed and the lesion of interest showed high activity in the blood-pool phase. What is the most likely diagnosis?

a Chondroma

b Simple bone cyst

c Aneurysmal bone cyst

d Chondromyxoid fibroma

e Enostosis (bone island)

A

58 Answer C: Aneurysmal bone cyst

An aneurysmal bone cyst is very vascular hence exhibits increased uptake in the blood-pool phase. A simple bone cyst has no uptake. An enostosis has no uptake.

159
Q

59 A 34-year-old man with arm pain had a radiograph which showed a permeative lesion in his humerus, which was expansile and associated with a soft-tissue mass and moderate amount of lamellated periosteal reaction. What is the most likely cause?

a Simple bone cyst

b Lymphoma of bone

c Chondroblastoma

d Multiple myeloma

e Non-ossifying fibroma

A

59 Answer B: Lymphoma of bone

lymphoma is most likely to have this appearance; it is relatively slow growing. Multiple myeloma is lucent but has no expansion. A sarcoma would also be possible but is not listed.

160
Q

(Ped) 60 A 10-year-old girl presents with a long history of right hip pain. A plain radiograph shows a lucent lesion in the greater trochanter. It has sclerotic margins and contains central irregular calcifications and is associated with a thick periosteal reaction. Which of the following is the most likely diagnosis?

a Enchondroma

b Aneurysmal bone cyst

C Giant cell tumour

d Chondroblastoma

e Chondromyxoid fibroma

A

60 Answer D: Chondroblastoma

Chondroblastoma is a benign cartilaginous tumour that normally occurs before closure of the growth plates and is characteristically found in the epiphyses of long bones, particularly the femur and tibia. Patients tend to present with a long history of pain. They may become locally aggressive and rarely metastasise.

161
Q

61 At what age does a haemangiopericytoma most commonly present?

a 0-5 years

b 10-20 years

C 30-40 years

d 50-60 years

e Over 70 years

A

61 Answer C: 30-40 years

Haemangiopericytoma is a rare tumour that usually occurs in the soft tissue of the thigh, pelvis or retroperitoneum and presents as a painless slow-growing mass. They may show locally aggressive behaviour.

162
Q

62 An 18-year-old female presented with hip pain and was found to have a welldefined lesion in her greater trochanter, which was ultimately proven to be a chondroblastoma. What pattern of mineralisation is typical?

a Normal surrounding bone, no mineralisation in lesion

b Normal surrounding bone, rings and arcs within lesion

C Surrounding sclerosis, no mineralisation in lesion

d Surrounding osteopenia, dense sclerosis within lesion

e Surrounding osteopenia, popcorn-shaped densities within lesion

A

62 Answer B: Normal surrounding bone, rings and arcs within lesion

163
Q

(Ped) 62 A radiograph of the femur in a 10-year-old shows an ill-defined mixed sclerotic/ lytic metaphyseal lesion in the distal femur. There is cortical disruption with a sunburst periosteal reaction and an associated soft-tissue mass containing speckled calcification. Which of the following is the most likely diagnosis?

a Ewing’s sarcoma

b Osteosarcoma

C Osteoid osteoma

d Osteoblastoma

e Osteochondroma

A

62 Answer B: Osteosarcoma

Osteosarcoma usually presents with a one- to two-month history of painful swelling and often with associated fever. Ewing’s sarcoma rarely contains any calcification.

164
Q

70 A 3 7-year-old in ale had a CT of his pelvis to investigate abdominal pain and was found to have appendicitis. An incidental finding of a 7-mm enostosis (bone island) was made in his ilium. What is the most appropriate follow-up for the enostosis?

a None

b Chemotherapy

c Radiofrequency ablation

d Excision

e Follow up CT in one year

A

70 Answer A: None

Enostoses are benign incidental findings that may increase slowly or decrease in size. It could be significant if at a critical load-bearing region but at 7 mm it is unlikely to be clinically relevant in the ilium.

165
Q

5 A seven-year-old boy who is known to have Ollier’s disease presents with a solid lump close to his knee. What radiological finding is likely?

a Cartilaginous rest

b Endosteal reaction

C Haemangioma

d Lymphangioma

e Bony spurs pointing away from the joint

A

5 Answer A: Cartilaginous rest

Cartilaginous rests are round radiolucencies, caused by displaced cartilage in the bone. An enchondroma is a benign cartilaginous lesion. Enchondromatosis, also known as Ollier’s disease, is due to derangement of cartilaginous growth, leading to migration of cartilaginous rests from the epiphysis to the metaphysis, where proliferation occurs. It is usually diagnosed in childhood and, in addition to abnormal growth, it is associated with juvenile granulosa cell tumours of the ovary. The main complication is sarcomatous transformation, which is more common in axial rather than appendicular enchondromas. Maffucci’s syndrome is similar except that it also involves haemangiomas and lymphangiomas.

166
Q

6 A 70-year-old man is known to have histologically confirmed prostate cancer following an ultrasound-guided biopsy a month ago. His PSA has risen to 42 ng/mL and he presents to clinic with increasing lower back pain but no neurology. Which of the following is the single best form of appropriate imaging?

a Plain film skeletal survey

b Radionuclide whole body bone scan

C CT whole spine

d Contrast enhanced MR of the lumbar spine and pelvis

e Three-phase radionuclide bone scan of the lumbar spine and pelvis

A

6 Answer B: Radionuclide whole body bone scan

Skeletal survey is more appropriate for multiple myeloma screening. Some literature states PSA<20 does not warrant a bone scan although centres vary in their practice. CT of the whole spine is unnecessary, although a targeted study can be useful if there is doubt. Similarly MR can be performed but this should be targeted at areas flagged up by a bone scan. 3-phase bone scans are used for targeted regions to assess for loosening and infection.

167
Q

(CNS) 11 A 14-year-old female exhibited painless facial asymmetry, which had been progressing since birth and for which she had undergone multiple surgical procedures with reconstruction. CT findings revealed expansion of the medullary cavity of the left maxillary bone with a `ground-glass’ appearance. The skull base was also imaged and the neural foramina were not involved. The lesion was hypointense on both Ti- and T2-weighted images. What is the most likely diagnosis?

a Paget’s disease

b Noonan’s syndrome

C Ossifying fibroma

d Fibrous dysplasia

e Osteogenesis imperfecta

A

11 Answer D: Fibrous dysplasia

Fibrous dysplasia is a pathology of unknown aetiology and usually occurs before 30 years of age. It involves increased proliferation of woven bone within the medullary cavity that cannot develop to lamellar bone and is most commonly monostotic but can be polyostotic. Bone expansion resulting in facial deformity can occur and malignant transformation has been described (<0.5 %). If the skull base is affected the neural foraminae can become stenosed. The radiological appearances are diverse dependent on the degree of fibrous tissue produced. A CT scan characteristically reveals an expansile bony lesion that displays groundglass density. The bony expansion can be identified as a result of widening of the medullary space with the cortex remaining intact. MRI usually shows a low signal intensity on both Ti- and T2-weighted images and intense enhancement following administration of contrast. Polyostotic fibrous dysplasia is a feature of Albright’s syndrome (cutaneous pigmentation, fibrous dysplasia and precocious puberty).
Paget’s disease produces mixed lyric and sclerotic lesions, is more common in males, primarily involves the vertebrae and skull and occurs later in life. Ossifying fibromas are more focal, displaying discrete zones of osseous or fibrous tissue and behaving more aggressively.

168
Q

15 A 60-year-old patient with hip pain was assessed with a radiograph of their pelvis. Small lucent lines at right angles to the cortex on the medial border of the proximal femoral shaft were visible bilaterally. What would be the most likely underlying diagnosis?

a Osteoporosis

b Fibrous dysplasia

C Hyperthyroidism

d Intraosseous desmoid

e Achondroplasia

A

15 Answer B: Fibrous dysplasia

There are at least 10 causes but the four most common are: osteomalacia, fibrous dysplasia, osteogenesis imperfecta, Paget’s. Pseudofractures are the same as Looser lines or osteoid seams, and are insufficiency fractures and non-union. They occur at sites of mechanical stress such as where vessels enter bone. They tend to be bilateral and symmetric at right angles to the bone margin. The pathognomonic sign is a 2-3 mm stripe of lucency at right angles to the cortex (osteoid seam).

169
Q

(CNS) 21 A 60-year-old woman presents with a painless mastoid swelling. Investigations reveal a mixed hearing loss. CT shows a coarsening of the trabeculae with cortical expansion and thickening. What is the most likely diagnosis?

a Paget’s disease

b Osteomyelitis of skull base

C Ossifying fibroma

d Polyostotic fibrous dysplasia

e Monostotic fibrous dysplasia

A

Answer A: Paget’s disease

Paget’s disease presents with a mixed hearing loss due to fixation of the stapes in the oval window, giving a conductive deficit and loss of cochlear bone density, producing a sensorineural deficit. Both monostotic and polyostotic fibrous dysplasia normally present as an incidental finding in a young age group with the classical ground-glass appearance on CT. The clinical picture does not fit with a potential diagnosis of osteomyelitis. Typical CT appearances of ossifying fibroma are of a cortically based lyric lesion with a thick bony rim in a young patient.

170
Q

(Ped) 35 A 12-year-old boy presents with long-standing left knee pain with no history of trauma. A plain radiograph shows a lucent lesion within the metaphysis of the proximal tibia. This has a welldefined sclerotic margin and contains trabeculations. There is no associated periosteal reaction. Which of the following is the most likely diagnosis?

a Enchondroma

b Chondromyxoid fibroma

C Simple bone cyst

d Non-ossifying fibroma

e Fibrous dysplasia

A

35 Answer B: Chondromyxoid fibroma

Chondromyxoid fibroma is a rare benign cartilaginous tumour that presents with pain. It is usually found in the metaphysis of the long bones, most commonly in the proximal tibia. There is no associated periosteal reaction unless it is fractured.

171
Q

36 An 83-year-old man was admitted to hospital with a chest infection. Blood tests showed a markedly elevated serum alkaline phosphatase, low albumin, elevated calcium and reduced phosphate. A radiograph of his lumbar spine showed multiple poorly defined sclerotic lesions. What is the most likely underlying diagnosis?

a Fibrous dysplasia

b Ewing sarcoma

C Osteomalacia

d Osteochondromatosis

e Prostate carcinoma

A

36 Answer E: Prostate carcinoma

Elevated alkaline phosphatase in elderly men must raise the suspicion of bony metastases from prostate carcinoma, which is the commonest cause of sclerotic metastases.

172
Q

37 A 43-year-old gentleman presented with hyperglycaemia and left-sided chest pain over a period of four months. A radiograph of his chest demonstrated a mass centred on a rib destroying the overlying cortex and associated with a soft-tissue mass. The hyperglycaemia was investigated and felt to be a paraneoplastic phenomenon. What is the most likely diagnosis?

a Adamantinoma

b Chondrosarcoma

C Osteosarcoma

d Myeloma

e Ewing sarcoma

A

37 Answer B: Chondrosarcoma

Both chondrosarcoma and osteosarcoma commonly present with paraneoplastic hyperglycaemia (85% of cases with central chondrosarcoma, 25% of osteosarcoma) but the patient’s age and the site are more typical for a chondrosarcoma.

173
Q

38 A 30-year-old man is found to have an incidental abnormality in his distal femur after a radiograph was performed to investigate a skiing injury. This is a 4-cm well-defined lucent lesion at the lateral margin of the distal end of the femur abutting the articular surface without expansion. What is the most likely diagnosis?

a Aneurysmal bone cyst

b Enchondroma

c Osteoblastoma

d Giant cell tumour

e Osteochondroma

A

38 Answer D: Giant cell tumour

Giant cell tumours are almost invariably epiphyseal although they may extend to involve the metaphysis. They occur almost exclusively in those with closed epiphyses and are eccentric with a non-sclerotic margin.

174
Q

(Ped) 38 A 15-year-old girl complaining of long-standing right-sided chest pain has a CXR. This shows a right-sided pleural effusion and an abnormality of the sixth right anterior rib. There is a 4-cm illdefined lytic lesion within the rib associated with a very large inhomogeneous soft-tissue mass. Which of the following is the most likely diagnosis?

a Fibrous dysplasia

b Enchondroma

C Ewing’s sarcoma

d Haematopoiesis

e Aneurysmal bone cyst

A

38 Answer C: Ewing’s sarcoma

Forty per cent of Ewing’s sarcoma are found in the flat bones and usually have a disproportionately large soft-tissue mass. They have a large intrathoracic component and a minimal extrathoracic component.

175
Q

39 A 33-year-old previously healthy man presented with a two-month history of bone pain and headaches. Blood tests show a serum calcium of 13.2 mg/ dL with phosphate of 0.6 mM (reference ranges: calcium 8.5-10.5 mg/dL, phosphate 0.8-1.5 mM). Which of the following bone lesions is most frequently associated with these findings?

a Osteitis fibrosa cystica

b Fibrous dysplasia

C Osteochondroma

d Paget’s disease of bone

e Giant cell tumour

A

39 Answer A: Osteitis fibrosa cystica

Osteitis fibrosa cystica is caused by an excess of PTH causing increased osteoclastic resorption of bone. It is seen in people with hyperparathyroidism and can present with headaches and bone pain.
Fibrous dysplasia, osteochondroma, Paget’s disease of bone and giant cell tumour are not associated with disorders of calcium metabolism.

176
Q

40 A 24-year-old university student is awaiting treatment of a chondromyxoid fibroma in his proximal tibia when he presents to a different hospital with an incidental injury to his leg. No fracture is visible but what is the expected appearance of this tumour?

a Well-defined lucent lesion with a non-sclerotic margin in the epiphysis

b Well-defined lucent lesion with sclerotic rim in the metaphysis

C Poorly defined lucent lesion in the diaphysis

d Poorly defined lucent lesion in the epiphysis

e Poorly defined lucent lesion in the metaphysis

A

40 Answer B: Well-defined lucent lesion with sclerotic rim in the metaphysis

Chondromyxoid fibromas generally occur between the ages of 10 and 30 years. Internal calcification is uncommon.

177
Q

42 A 25-year-old man was found to have a giant cell tumour in his tibia. What location in the bone is most typical?

a Epiphyseal

b Epiphyseal and metaphyseal

C Metaphyseal

d Metaphyseal and diaphyseal

e Diaphyseal

A

42 Answer A: Epiphyseal

Giant cell tumours are usually epiphyseal in a sub-articular location but may extend from the epiphysis to involve the metaphysis. Ninety-seven per cent occur after closure of the epiphyses.

178
Q

(Ped) 42 A 15-year-old boy presents with a two-month history of dull thoracic back pain that is worse at night. On examination there is a scoliosis concave to the right. A radiograph of the thoraco-lumbar spine confirms the scoliosis and shows a lucent lesion originating in the spinous process of T8 extending into the T8 vertebral body. Which of the following is the most likely diagnosis?

a Osteoblastoma

b Osteochondroma

c Osteoid osteoma

d Haemangioma

e Fibrous dysplasia

A

42 Answer A: Osteoblastoma

Approximately half of osteoblastomas are found in the spine, most of which arise in the posterior elements. They typically present with painful scoliosis, worse at night.

179
Q

43 A 73-year-old male with a history of Paget’s disease presented with insidious onset low back pain and was assessed with radiographs and MRI, CT and bone scans. Radiographs showed a mixed lytic/sclerotic appearance in the sacrum and an MRI showed infiltration of all five sacral segments with additional anterior soft-tissue extension. There was evidence of osteoid mineralisation within this mass on CT. A bone scan showed these were the only hotspots. What is the most likely diagnosis?

a Sclerotic metastasis

b Lymphoma

c Ewing sarcoma

d Osteoblastoma

e Osteosarcoma

A

43 Answer E: Osteosarcoma

Osteosarcoma may rarely arise in Paget’s disease through malignant degeneration. Sclerotic metastases are probably more common even in someone known to have Paget’s but are usually multifocal and would rarely involve only one part of one bone.

180
Q

44 A 22-year-old male with an osteosarcoma of his tibia underwent a staging CT which demonstrated a small nodule in his left lower lobe that was thought to represent a metastasis. What is the prognosis for patients with this clinical picture? (Options given as percentage surviving five years from diagnosis)

a 60-80%

b 40-50%

c 30-40%

d 20-30%

e 10-20%

A

44 Answer E: 10-20%

Osteosarcoma has a very poor prognosis once metastases are present.

181
Q

45 A 20-year-old man slowly developed predominantly nocturnal low back pain. He reports that salicylate-based medication is particularly helpful in relieving his pain. What is the most likely diagnosis?

a Fibrosarcoma

b Malignant fibrous histiocytoma

c Osteoid osteoma

d Chondrosarcoma

e Ollier’s disease

A

45 Answer C: Osteoid osteoma

Osteoid osteoma is a benign bone tumour composed of osteoid and woven bone that usually affects young individuals. They are small tumours (less than 2 cm in diameter) with a central nidus in which prostaglandin E2 is elevated. This has been suggested to be the cause of the pain experienced and explains the typical history of relief with salicylates.

182
Q

47 An 11-year-old boy presented with left-sided groin pain and recent weight loss. He had been finding it particularly difficult getting in and out of the car when going to school. His mother had been giving him aspirin that helped initially but this no longer had much of an effect. Plain radiographs revealed a 9-mm well-circumscribed lesion with a sclerotic border and central radiolucent nidus within the femoral neck. What is the most likely diagnosis?

a Aneurysmal bone cyst

b Fibrous dysplasia

c Stress fracture of the femoral neck

d Brodie’s abscess

e Osteoid osteoma

A

47 Answer E: Osteoid osteoma

The femoral neck is the most common location for osteoid osteomas.

183
Q

48 A 34-year-old man presented with a two-month history of pain in his right knee. On examination his right knee was enlarged and tender. A radiograph showed a 9-cm lytic lesion in the epiphysis of the distal femur with a `soap bubble’ appearance. A biopsy showed multinucleated cells in a stroma with spindle-shaped mononuclear cells. What is the most likely diagnosis?

a Tuberculosis

b Giant cell tumour

c Osteosarcoma

d Chondrosarcoma

e Malignant fibrous histiocytoma

A

48 Answer B: Giant cell tumour

Giant cell tumours are typically epiphyseal and the `soap bubble’ appearance is characteristic. More destruction would be expected with the other options.

184
Q

49 A bone scan of a 17 year old shows a `double-density’ sign in the femoral neck with a focal centralised region of intense activity surrounded by a region of lesser activity. What is the most likely cause?

a Brodie’s abscess

b Simple bone cyst

c Bone infarction

d Osteoid osteoma

e Stress fracture of femoral neck

A

49 Answer D: Osteoid osteoma

Osteoid osteoma has a typical double-density sign of a nidus with surrounding activity due to the reactive sclerosis. With a stress fracture of the femoral neck it is unusual to see the double-density sign. Brodie’s abscess usually has a more uniform pattern of radiotracer uptake. A simple bone cyst has no uptake on a bone scan.

185
Q

50 A patient with a known area of fibrous dysplasia in their femur subsequently presented with a soft tissue mass in their thigh and an MRI was arranged. The mass was of low signal on Ti- and high signal on T2-weighted images and a biopsy showed it to be a myxoma. What is the likely diagnosis?

a Mazabraud’s syndrome

b Ollier’s syndrome

C Maffucci’s syndrome

d Metastatic malignancy

e Gardner’s syndrome

A

50 Answer A: Mazabraud’s syndrome

Mazabraud’s syndrome is the rare association of fibrous dysplasia with intramuscular myxoma and is an important differential for a soft-tissue mass with lytic bone lesions.

186
Q

51 Based on age alone, what is the most likely primary bone tumour in a 45-yearold male?

a Osteosarcoma

b Chondrosarcoma

c Giant cell tumour

d Aneurysmal bone cyst

e Osteoid osteoma

A

51 Answer B: Chondrosarcoma

Chondrosarcoma tends to occur in those over 40 and the median age at presentation is 45 years. Giant cell tumour has a peak incidence in the third and fourth decades, and has a female preponderance. Osteosarcoma is most often seen in the first to third decades of life, although there is a second peak after the age of 60 due to Paget’s disease. Aneurysmal bone cyst is most commonly present in the second and third decades and osteoid osteoma in the first three decades.

187
Q

52 A 15-year-old boy was recently diagnosed with Ewing sarcoma. What is the most likely bone to be affected?

a Femur

b Humerus

C Sacrum

d Scapula

e Tibia

A

52 Answer A: Femur

The most common location is the femur followed by the ilium, tibia, humerus, fibula sacrum and ribs in that order.

188
Q

53 A 14-year-old girl was recently diagnosed with a Ewing sarcoma of her left femur. Which part of the bone is most likely to be affected?

a Epiphyseal

b Epiphyseal and metaphyseal

C Metaphyseal

d Metaphyseal and diaphyseal

e Diaphyseal

A

53 Answer D: Metaphyseal and diaphyseal

Approximately two-thirds of Ewing sarcomas affect long bones and half of these are centred in themetadiaphysis

189
Q

54 A 16-year-old boy noticed pain in his knee during his weekly football practice, which was gradually increasing in severity over months. A radiograph showed an aggressive lesion causing a moth-eaten appearance of bone destruction and an associated soft-tissue mass centred on the distal femur. What is the most likely diagnosis?

a Chondrosarcoma

b Metastasis

C Osteochondroma

d Osteogenic sarcoma

e Synovial sarcoma

A

54 Answer D: Osteogenic sarcoma

Osteogenic sarcoma is the most common primary malignant bone tumour. It classically affects adolescents with a second peak in incidence after the age of 60 due to Paget’s disease. The typical presentation is with pain of several weeks’ duration, commonly with activity. The most common bones involved are the femur and tibia with the majority of lesions occurring around the knee. An important differential is osteomyelitis.

190
Q

55 A six-year-old boy was brought to the Emergency Department after falling from a climbing frame. A radiograph showed a fallen fragment within a well-defined unilocular lesion in the proximal humeral metaphysis. There was cortical thinning but no periosteal reaction. What is the most likely underlying diagnosis?

a Osteitis deformans

b Engelmann’s disease

C Fibrous dysplasia

d Simple bone cyst

e Osteoid osteoma

A

55 Answer D: Simple bone cyst

191
Q

56 A radiograph of a 16 year old shows a well-defined lytic lesion with a thin sclerotic margin in the epiphysis of the proximal tibia. On MRI, STIR and T2-weighted images demonstrate a high signal, lobulated lesion with a thin low signal margin. Extensive surrounding bone oedema is seen. What is the most likely diagnosis?

a Aneurysmal bone cyst

b Giant cell tumour

C Chondrosarcoma

d Chondroblastoma

e Brodie’s abscess

A

56 Answer D: Chondroblastoma

Chondroblastomas are usually spherical, well-defined lesions in an epiphyseal location and with a fine sclerotic margin. Florid marrow oedema surrounding the lesion is also a typical feature. Aneurysmal bone cysts have a low signal rim on both T1- and T2 -weighted MRI images but bone marrow oedema is not typical. Giant cell tumours are rare in children and their margins are not sclerotic. Chondrosarcomas are epiphyseal but rare in this age group.

192
Q

57 A 15-year-old patient felt a lump on their hand and a radiograph demonstrated an enchondroma. From which part of the bone do enchondrornas develop?

a Epiphysis

b Epiphysis and metaphysis

C Metaphysis

d Metaphysis and diaphysis

e Diaphysis

A

57 Answer C: Metaphysis

Enchondromas are metaphyseal, although they may affect the epiphysis after closure of the growth plate.

193
Q

(Ped) 57 A 14-year-old boy presented with history of aching leg pain for a few months that was more severe at night and relieved by non-steroidal anti-inflammatory drugs. A lateral radiograph of his leg showed a <1 cm lucent nidus surrounded by dense sclerosis within the diaphysis of the tibia and an osteoid osteoma is suspected. A CT is planned for further evaluation. What are the expected findings on the CT scan?

a A well-demarcated low-attenuation lesion a few millimetres in size, surrounded by an area of high attenuation

b A localised low-attenuation lesion, which does not enhance with contrast with surrounding high attenuation and a small channel to the surrounding soft tissues

c Well-demarcated low-attenuation area of 2.5 cm in size, surrounded by an area of high attenuation

d A high-attenuation lesion with no area of low attenuation

e A linear high-attenuation lesion

A

57 Answer A: A well-demarcated low-attenuation lesion a few millimetres in size, surrounded by an area of high attenuation

Osteoid osteoma is a common benign bone tumour. Its commonest location is within the cortex of long bones. It characteristically presents with pain that is worse at night and is relieved by non-steroidal anti-inflammatory drugs. On plain radiographs it is a small (<1.5 cm) lytic lesion, which is surrounded by reactive sclerosis. The nidus may not be visible on plain radiographs due to the density of the surrounding sclerosis. CT shows the nidus in these cases. Osteoblastoma is a rare benign tumour that can undergo malignant transformation. It is similar histologically to osteoid osteoma. It has a nidus of >1.5 cm that ranges from lytic to densely sclerotic in appearance.

194
Q

58 A female neonate was born prematurely after a pregnancy complicated by polyhydramnios. A USS of the sacral region demonstrated an 18-cm solid cystic mass and pelvic X-ray demonstrated a mass with amorphous calcification. What blood test would be most helpful in making the diagnosis?

a Beta HCG

b Full blood count

c Alpha feta protein

d Liver function tests

e Urea and electrolytes

A

58 Answer C: Alpha feta protein

Sacrococcygeal teratoma is the most common solid tumour occurring in the newborn and alpha feta protein is raised in malignant teratomas. It is more common in girls and is associated with other congenital anomalies such as spinal dysraphism, sacral agenesis, hydronephrosis, imperforate anus and gastroschisis. Sacrococcygeal teratomas are typically mixed cystic/solid and on MRI they are heterogeneous with high signal on T1-weighted images. The older the child is at presentation the more likely the tumour is to be malignant.

195
Q

59 A 55-year-old man had a history of renal cell carcinoma treated with a radical nephrectomy 10 months previously (T4N1M0). He was thought to be in remission but developed bony pain in his lumbar spine. Lumbar radiographs revealed an abnormality that showed increased uptake on a subsequent Tc-99MDP bone scan. What is the most likely appearance of this lesion on the plain film?

a Expansile, lytic lesion

b Non-expansile, lytic lesion

C Expansile, sclerotic lesion

d Non-expansile sclerotic lesion

e Non-expansile, mixed sclerotic/lytic lesion

A

59 Answer A: Expansile, lytic lesion

Thyroid and renal cell carcinoma metastases are nearly always osteolytic. Other causes of lytic metastases include melanoma, lung and breast carcinomas. The most common causes of sclerotic metastases are prostatic, breast, colonic and bladder carcinomas in addition to melanoma and soft tissue sarcomas.

196
Q

62 Where in the body are enchondroma most likely to occur?

a Femur

b Humerus

C Metacarpals

d Ribs

e Tibia

A

62 Answer C: Metacarpals

The majority occur in the small tubular bones of the hand.

197
Q

63 At what age is an osteoid osteoma most likely to present?

a 0-10 years

b 10-30 years

c 30-50 years

d 50-70 years

e Over 70 years

A

63 Answer B: 10-30 years

Three-quarters will be within this range

198
Q
  1. A 34-year-old sedentary male office worker presents with a 2-month history of heel pain. A radiograph demonstrates a well-defined lytic lesion in the calcaneum. This produces mild expansion with endosteal scalloping and has a central ossified nodule. On MRI, the lesion is high signal on T1 and T2. What is the most likely diagnosis?

(a) Giant cell tumour

(b) Fibrous cortical defect

(c) Intraosseous lipoma

(d) Osteoid osteoma

(e) Solitary bone cyst

A
  1. (c) Intraosseous lipoma

This is an expansile, non-aggressive lesion usually seen in metaphyses. It may contain a focal area of dystrophic calcification within (secondary to fat necrosis). On MRI, the lesion returns fat signal on all sequences (high signal on T1 and T2, and low signal on STIR).

199
Q
  1. A 10-year-old boy presented with fracture of the left proximal humerus sustained during ossificans ossificans a tackle in a football match. Plain radiographs show a pathological fracture and underlying lytic lesion in the metaphysis of the proximal humerus. The lesion shows endosteal scalloping and a small bone fragment in the floor of the cyst. MRI features include intermediate signal on T1 and high signal on T2 with a fluid–fluid level. What is the most likely diagnosis of the underlying bony lesion?

(a) Unicameral bone cyst

(b) Lymphoma

(c) Aneurysmal bone cyst

(d) Telangiectatic osteosarcoma

(e) Giant cell tumour

A
  1. (a) Unicameral bone cyst

This is usually asymptomatic unless fractured. The lesion usually is 2–3 cm in size, usually in the metaphysis, with its long axis parallel to the long axis of the bone. There may be endosteal scalloping of the bone and a ‘fallen-fragment sign’ (if fractured, centrally dislodged fracture fragment falls into a dependent position in the cyst).

200
Q
  1. A 15-year-old boy attended the Accident & Emergency Department with anklepain after a twisting injury 7 days previously. The history suggests there has been ill-defined swelling and ache for a few weeks. The plain radiograph shows a fracture in the distal fibula, with lamellar periosteal reaction. There appears to be an associated soft tissue bulge. What is the most likely diagnosis?

(a) Fracture with large haematoma

(b) Neuroblastoma metastasis

(c) Lymphoma

(d) Ewing’s sarcoma with fracture

(e) Osteomyelitis

A
  1. (d) Ewing’s sarcoma with fracture

For a simple fracture with haematoma, this case has presented too early for a periosteal reaction. Neuroblastoma metastasis could be considered in a child less than 5 years old, and lymphoma should be considered in patients over 30 years age. Osteomyelitis could have been possible if there had been a previous history of localised pain, fever etc.

201
Q
  1. A 20-year-old athlete presented with chronic leg pain relieved with aspirin. The plain radiograph shows 2–3 cm area of sclerosis and cortical thickening in the midshaft tibia. CT shows a 1 cm lytic lesion with central mineralisation. MRI demonstrates a bone oedema pattern but normal surrounding soft tissues. The most likely cause of underlying pathology is?

(a) Stress fracture

(b) Ewing’s sarcoma

(c) Osteomyelitis

(d) Enchondroma

(e) Osteoid osteoma

A
  1. (e) Osteoid osteoma

This typically presents as a small lytic lesion with central mineralisation surrounded by reactive sclerotic area. On bone scans, the central nidus shows intense uptake
surrounded by region of lesser activity (double density sign). MRI demonstrates bone oedema pattern and typically relieved with aspirin. Stress fractures are transverse and linear. Ewing’s sarcoma and infection must be considered if soft tissue involvement is seen on MRI.

202
Q
  1. A 40-year-old mother presents with knee pain. Plain radiographs show an eccentric, expansile, lytic lesion with a narrow zone of transition in the lateral femoral condyle extending to subarticular bone. The lateral cortex is thinned but no periosteal reaction or sclerosis is seen. MRI shows a well-defined bony lesion with intermediate signal on T1 and mixed signal on T2 and multiple fluid–fluid levels. The most likely diagnosis is?

(a) Benign fibrous histiocytoma

(b) Giant cell tumour

(c) Telangiectatic osteosarcoma

(d) Brodie’s abscess

(e) Simple bone cy

A
  1. (b) Giant cell tumour

Typical features on radiograph are a lytic, eccentric, expansile, subarticular lesion with ‘soap bubble’ appearance (fluid–fluid level on MRI). Benign fibrous histiocytoma and telangiectatic osteosarcoma can also show fluid– fluid levels but are metaphyseal lesions not extending to the subarticular surface. A simple bone cyst is unilocular and situated away from the articular surface.

203
Q

(Ped) 12. A 10-year-old child presents with a lump in the scalp. The skull radiograph shows a lucent lesion with sclerotic margins. The most likely diagnosis is?

(a) Dermoid cyst

(b) Aneurysmal bone cyst

(c) Histiocytosis X

(d) Neuroblastoma metastasis

(e) Osteosarcoma metastasis

A
  1. (a) Dermoid

They are usually an incidental finding. They have a characteristic appearance of a central lucent area with sclerotic margins.

204
Q

(Ped) 13. A 13-year-old child presents with pain in the leg. Radiography shows a welldefined, eccentric, radiolucent lesion with a thin sclerotic border towards the medulla in the proximal tibia. No periosteal reaction seen. On MRI, the lesion returns low signal on T1 and T2. The most likely diagnosis is?

(a) Chondromyxoid fibroma

(b) Non-ossifying fibroma

(c) Intraosseous ganglion

(d) Brodie’s abscess

(e) Simple bone cyst

A
  1. (b) Non-ossifying fibroma or a fibrous cortical defect

These benign lesions typically present like this. On MRI, they appear as low signal on T1 and T2 images due to hypocellular fibrous tissue within. Chondromyxoid fibromas show a bulging cortex and geographical bone destruction with calcifications and septations. On MRI they are hyperintense on T2. Intraosseous ganglion is hyperintense on T2. Aneurysmal bone cysts show fluid–fluid levels and are heterogenously hyperintense on MRI.

205
Q
  1. A 21-year-old man presents with multiple swellings and focal areas of bluish discoloration in both hands. Plain radiograph of the hands show multiple, welldefined, expanded lytic lesions in the metacarpals. These lesions show stippled calcifications in the matrix and cause cortical thinning. Multiple small round calcifications are seen in the surrounding soft tissues. The most likely diagnosis is?

(a) Ollier’s disease

(b) Maffucci’s syndrome

(c) Metastases

(d) Diaphyseal aclasis

(e) Kaposi’s sarcoma

A
  1. (b) Maffucci’s syndrome

This diagnosis is a combination of multiple enchondromatosis and soft tissue haemangiomas. Hand and foot involvement is common and severe. The soft tissue calcifications are phleboliths from haemangiomas. In Ollier’s disease, there is absence of haemangioma and phleboliths.

206
Q

(Ped) 19. A 14-year-old child presents with left hip pain for 6 months. Radiography shows a well-circumscribed lytic lesion in the greater trochanter with a sclerotic margin and matrix calcifications. CT confirms the findings of the radiograph. On MRI, the lesion returns low signal on T1 and high on T2, with surrounding marrow and soft tissue oedema. The most likely diagnosis is?

(a) Giant cell tumour

(b) Chondroblastoma

(c) Epiphyseal osteomyelitis

(d) Langerhans cell histiocytosis

(e) Aneurysmal bone cyst

A
  1. (b) Chondroblastoma

This is the most common neoplasm seen in the apophysis/epiphysis of skeletally immature patients. These are typical imaging features, showing a lytic lesion with sclerotic margins and calcifications in the matrix. There is surrounding marrow and soft tissue oedema. MRI overestimates the extent. The tumour is almost always benign but may become locally aggressive.

207
Q
  1. A 35-year-old woman presents with swelling in the thigh. The radiograph shows a bony excrescence from the femoral cortex without medullary continuity. On MRI there is a soft tissue surrounding the bony excrescence, which returns high signal on T1 and T2 and homogenous low signal on STIR. The most likely diagnosis is?

(a) Osteochondroma

(b) Osteosarcoma

(c) Liposarcoma

(d) Parosteal lipoma

(e) Intramuscular lipoma

A
  1. (d) Parosteal lipoma

These are benign tumours of adipose tissue which are intimately related to the periosteum. They often contain bony excrescences that may resemble osteochondroma but, unlike osteochondroma, they do not communicate with the medullary cavity of parent bone. MRI is diagnostic, confirming the juxtacortical benign nature of the fatty lesion and non-communication of the bony lesion with the medulla of the adjacent bone.

208
Q
  1. A 12-year-old boy presents with a hard lump around his right knee. A radiograph shows a bony projection from the medial part of the tibial metaphysis with continuity of the cortex and medulla of the tibia. What is the most likely diagnosis?

(a) Osteochondroma

(b) Parosteal osteosarcoma

(c) Chondrosarcoma

(d) Periosteal osteosarcoma

(e) Juxtacortical myositis ossificans

A
  1. (a) Osteochondroma

The best diagnostic feature of osteochondroma is continuity of the bony cortex and medulla with the parent bone.

209
Q
  1. A 20-year-old woman presents with pain after injury to the index finger. The radiograph shows a 2 cm lytic lesion with a matrix containing calcifications. There is endosteal scalloping of the cortex with cortical expansion, but no cortical breach. The most likely diagnosis is?

(a) Bone infarct

(b) Enchondroma

(c) Chondrosarcoma

(d) Juxtacortical chondroma

(e) Epidermoid cyst

A
  1. (b) Enchondroma

Radiographic features are typical and diagnostic. A bone infarct appears as a serpiginous area with sclerotic margins and no endosteal scalloping. Chondrosarcoma shows periosteal reaction and an associated soft tissue mass.

210
Q
  1. A 26-year-old man presents with dull pain in the left thigh not relieved with salicylates. Radiograph shows a 3 cm expansile lytic lesion in the mid-shaft of the left femur, which shows reactive sclerosis and periosteal reaction. The bone scan shows intense tracer uptake. The most likely diagnosis is?

(a) Osteoid osteoma

(b) Osteoblastoma

(c) Osteosarcoma

(d) Osteomyelitis

(e) Aneurysmal bone cyst

A
  1. (b) Osteoblastoma

Also called giant osteoid osteoma when measuring > 2 cm. Osteoid osteoma are < 2 cm, have a predilection for the axial skeleton and usually respond to salicylates. Osteosarcomas show cortical destruction, mineralised matrix and soft tissue mass. Aneurysmal bone cysts show fluid–fluid levels and no matrix calcification.

211
Q
  1. A 33-year-old man presents with a 2-year history of a hard lump on the left middle finger. A radiograph shows a 2 cm, well-defined, round, densely sclerotic lesion attached to the cortex of the proximal phalanx of the left middle finger. No cortical erosion or periosteal reaction is seen. A bone scan shows no tracer uptake The most likely diagnosis is?

(a) Enostosis

(b) Osteoma

(c) Parosteal osteosarcoma

(d) Osteochondroma

(e) Myositis ossificans

A
  1. (b) Osteoma

The best diagnostic clue is the densely sclerotic, well-defined lesion attached to the parent bone. Latent lesions show no tracer uptake.

212
Q

(Ped) 44. A 5-year-old boy presents with a 1-month history of pain in right leg. Radiography shows an ill-defined lucency in the proximal tibial metadiaphysis with periosteal reaction and a wide zone of transition. MRI shows a intramedullary lesion which returns intermediate signal on T1, high on STIR and has an extraosseous enhancing mass. What is the most likely diagnosis?

(a) Ewing’s sarcoma

(b) Osteomyelitis

(c) Enchondroma

(d) Fibrous dysplasia

(e) Giant cell tumour

A
  1. (a) Ewing’s sarcoma

More than 50% of cases are seen in long bones. They are common in the metadiaphyseal region and most common in the 5- to 10-year age group. Often presenting similarly to osteomyelitis. MRI is extremely useful in determining the extent of the tumour, which is low on T1, high on T2 and STIR and enhances with contrast.

213
Q

(CNS) 47. A 27-year-old woman with facial skin lesions presents with chronic hearing loss in the right ear. CT shows expansion of the petrous and mastoid bones with a ‘ground-glass’ appearance. What is the most likely diagnosis?

(a) Osteopetrosis

(b) Fibrous dysplasia

(c) Otosclerosis

(d) Paget’s disease

(e) Giant cell tumour

A
  1. (b) Fibrous dysplasia

CT appearances are typical showing increased volume of bone with ‘ground-glass’ appearance.

214
Q
  1. Which of the following are correct regarding intra-articular osteoid osteoma: (T/F)

(a) Most commonly affects the hip.

(b) Typically presents with nocturnal pain.

(c) The radiolucent nidus is surrounded by extensive reactive sclerosis.

(d) MRI is more specific than CT in detecting the nidus.

(e) Plain radiograph have a sensitivity exceeding 90% in detecting the nidus.

A

Answer:

(a) Correct
(b) Not correct
(c) Not correct
(d) Not correct
(e) Not correct

Explanation:
Nocturnal pain is a feature of long bone osteoid osteoma and not of intra-articular lesions. It is also less responsive to NSAIDs. Extensivereactive sclerosis surrounding the nidusis a feature of long bone lesion and not of intra-articular lesion. Nidus is often overlooked on radiographs. MRI is more sensitive than CT in detecting the nidus, however CT is more specific. MRI is sensitive as it is highly sensitive in demonstrating perilesional edema.

215
Q
  1. Which of the following are correct regarding giant cell tumours of bone (GCT): (T/F)

(a) Are typically subarticular in location.

(b) Metastasise in less than 10% of cases.

(c) Most commonly occur in the proximal humerus.

(d) Spinal GCTs affect primarily the posterior elements.

(e) Periosteal reaction is seen in 60-80% of cases.

A

Answer:

(a) Correct
(b) Correct
(c) Not correct
(d) Not correct
(e) Not correct

Explanation:
Most common location of GCT is around knee joint (50%-65%) with distal femur accounting for 23%-30% cases.
Proximal humerus accounts for 4-8% of the cases of GCT. Spinal GCT primarily affects the vertebral bodies with extension into the posterior elements. Vertebral body collapse is a frequent finding. Periosteal reaction is uncommon seen on radiograph only in 10-30% cases. Cortical penetration is however seen in 33-50% cases with associated soft tissue mass.

216
Q

(Ped) 7. Which of the following statements are correct? (T/F)

(a) Ewing’s tumors rarely arise within flat bones.

(b) Giant cell tumors of bone are most commonly seen in the 5-15 years age group.

(c) Parosteal osteosarcomas have a peak in the 10-20 years age group.

(d) Eosinophilic granuloma usually involves one bone only.

(e) Ewing’s tumors often demonstrate calcification on CT.

A

Answers:

(a) Not correct
(b) Not correct
(c) Not correct
(d) Correct
(e) Not correct

Explanation:

About 60% of Ewing tumors arise in long bones, most common site is metadiaphysis of femur. About 40% arise in flat bones especially in pelvis, particularly in patients over 20 years age. Calcification is rare in Ewing’s tumor of bone. It typically presents with an onion skin periosteal reaction on radiograph. Majority of giant cell tumors occur in patients following fusion of the epiphysis. Thus usually occur after 18-20 years of age. They have a narrow zone of transition and usually abut the articular margin. Parosteal osteosarcomas occur in an older age group than the periosteal type with 50% occurring after 30 years of age.

217
Q
  1. Which of the following are correct regarding osteosarcoma? (T/F)

(a) There is an increased incidence osteopetrosis.

(b) There an association with pineoblastoma.

(c) Telangiectatic osteosarcoma typically demonstrates fluid-fluid levels on MRI.

(d) It frequently metastasises to other bones.

(e) Parosteal osteosarcoma resembles myositis ossificans on radiograph.

A

Answers:

(a) Correct
(b) Correct
(c) Correct
(d) Not correct
(e) Correct

Explanation:
In osteosarcoma, metastatic spread is heterogeneous, as bones lack a lymphatic system. The lungs are the most frequent size, although bones may be involved. Lymphadenopathy occurs late in the disease and is a poor prognostic sign.

218
Q
  1. Features of diaphyseal aclasia (hereditary multiple exostosis) include: (T/F)

(a) Malignant transformation to chondrosarcoma occurs in 35-40 %

(b) Exostoses have a cap hyaline cartilage, often with a bursa formation over the cap.

(c) Exostoses arise from the metaphysis and point towards the joint

(d) Exostoses stop growing when the nearest epiphyseal centre fuses

(e) Autosomal recessive inheritance.

A

Answers:

(a) Not correct
(b) Correct
(c) Not correct
(d) Correct
(e) Not correct

Explanation:
Diaphyseal aclasia is an autosomal dominant disorder presenting between 2 to 10 years of age. The exostoses arise from the metaphysis of long bones and point away from the joint. Risk of malignant transformation to chondrosarcoma is less than 5 %.

219
Q
  1. Which of the following are correct regarding bone metastases (T/F)

(a) Metastases can be excluded in a patient with bone scintigraphy showing no abnormal uptake.

(b) Prostate metastases are always sclerotic.

(c) Metastases located in the medulla are o reduced signal on T1W and increased signal on fat suppressed T2W MRI images

(d) Identification of a ‘halo’ of high signal intensity around a lesion on T2W MRI suggests a benign lesion

(e) Melanoma metastases are usually lytic.

A

Answers:

(a) Not correct
(b) Not correct
(c) Correct
(d) Not correct
(e) Correct

Explanation:
Lesions which outgrow their blood supply appear photopenic on bone scintigraphy. Small percentage of prostatic metastases can be entirely lytic. ‘Halo’ of high signal around a lesion on T2WI is highly suggestive of metastases.

220
Q
  1. Which of the following are correct concerning enchondromas: (T/F)

(a) In the hand, diaphysis is the most common site

(b) Most frequent tumor found in the small bones of the hands.

(c) Calcification is rare

(d) MRI appearances are of low signal on T1W and high signal on T2W spine echo images

(e) Maffuci syndrome is characterised by multiple enchondromas and soft tissue cavernous haemangiomas

A

Answers:

(a) Correct
(b) Correct
(c) Not correct
(d) Correct
(e) Correct

Explanation:
Enchondromas are a relatively common benign medullary cartilaginous neoplasm, usually found in children or young adults which can lead to pathological fractures or undergo malignant degeneration. Mostly they are an incidental finding with benign imaging features.
Calcification especially chondroid type is common finding.

221
Q

(Ped) 28. Which of the following are correct regarding features of Ewing sarcoma: (T/F)

(a) Presentation with systemic sign and symptoms is recognized.

(b) Is usually located in the epiphyses

(c) Most commonly affects flat bones in children

(d) Onion skin periosteal reaction is a rare radiological finding.

(e) MRI demonstrates a soft tissue mass in 80-90% of patient

A

Answers:

(a) Correct
(b) Not correct
(c) Not correct
(d) Not correct
(e) Correct

Explanation:
Majority of Ewing sarcoma are located in diaphysis of the humerus, femur, tibia and fibula. Epiphysis is a rare location. In the first two decades lesions are predominantly located in long bones. After that time lesions in flat bones are more usual. Ewing is an aggressive tumor with permeative or moth eaten appearance on radiograph with lamellated, onion skin or spiculated periosteal reaction.

222
Q
  1. Which of the following are correct concerning aneurysmal bone cysts: (T/F)

(a) 60-80 % of aneurysmal bone cysts are found in under 20 year old.

(b) Periosteal reaction is a pathognomonic feature

(c) They demonstrate a fluid-fluid level on CT

(d) Hyper-vascularity is usually seen in the periphery of the lesion

(e) They have a recurrence rate of 20-30 % after surgical treatment

A

Answers:

(a) Correct
(b) Not correct
(c) Correct
(d) Correct
(e) Correct

Explanation:

Aneurysmal bone cysts are benign expansile tumour like lesion. Periosteal reaction is not seen until associated with a fracture.

223
Q

(CNS) 32. Which of the following are correct about fibrous dysplasia? (T/F)

(a) Facial bones are rarely involved.

(b) Exophthalmos is a recognised feature.

(c) There is an association with Cushing’s syndrome.

(d) In the calvaria, the frontal bone is most commonly involved.

(e) It effects the maxilla more frequently than the mandible.

A

Answers:

(a) Not correct
(b) Correct
(c) Correct
(d) Correct
(e) Correct

Explanation:
Fibrous dysplasia typically presents before the age of 30 years. The most common sites of involvement of the ribs, cranial, facial bones and femur and tibia.